Sie sind auf Seite 1von 28

CONSTI REVIEWER 1

PURPOSE AND EFFECT OF PREAMBLE- Aglipay vs Ruiz case INTERPRETATION/CONSTRUCTION OF THE CONSTITUTION - Francisco v HR

- This case is about Aglipay, Supreme Head of the Philippine Church, assailing the stamps Former President Joseph E. Estrada filed an impeachment complaint (first impeachment
produced and sold by the Director of Post that bears religious details (In commemoration complaint) against Chief Justice Hilario G. Davide Jr. and seven Associate Justices of this
with the 33rd International Eucharistic Congress which, allegedly, is an encroachment of Court for “culpable violation of the Constitution, betrayal of the public trust and other high
the separation of church and state principle, specifically violation of “the state shall not crimes.” The House Committee on Justice ruled that the first impeachment complaint was
promote a religion over others”. “sufficient in form,” but voted to dismiss the same for being insufficient in substance. Four
- The court ruled that the mere inclusion of the religious details does not constitute a months and three weeks since the filing of the first complaint, a day after the House
violation. In fact, what the act constitutes is for the improvement of the State and its Committee on Justice voted to dismiss it, the second impeachment complaint was filed by
economy through incorporating an international event plus the Philippines, being a Representatives Gilberto C. Teodoro, Jr. (First District, Tarlac) and Felix William B.
religious state as evident through its Preamble of the Constitution, (…. Imploring the Aid Fuentebella (Third District, Camarines Sur) against Chief Justice Hilario G. Davide, Jr., This
of Almighty God) second impeachment complaint was accompanied by a “Resolution of
- Constitution of the Philippines; Religious Freedom, Postage stamps issued under Endorsement/Impeachment” signed by at least one-third (1/3) of all the Members of the
act no. 4052 contemplates no religious purpose in view. House of Representatives.

SUPREMACY OF THE CONSTITUTION- Angara V. Electoral Commission Case If the plain meaning of the word is not found to be clear, resort to other aids is
available; The proper interpretation of a constitutional provision depends more on
This case is about Petitioner Jose Angara was proclaimed winner and took his oath of office how it was understood by the people adopting it than the framers’ understanding
as member of the National Assembly of the Commonwealth Government. On December 3, thereof. Statutory Construction; Verba Legis; Wherever possible, the words used in
1935, the National Assembly passed a resolution confirming the election of those who have the Constitution must be given their ordinary meaning except where technical terms
not been subject of an election protest prior to the adoption of the said resolution. On are employed. The words of the Constitution should be interpreted in accordance with
December 8, 1935, however, private respondent Pedro Ynsua filed an election protest the intent of the framers—ratio legis est anima—the object is to ascertain the reason
against the petitioner before the Electoral Commission of the National Assembly. The which induced the framers of the Constitution to enact the particular provision and the
following day, December 9, 1935, the Electoral Commission adopted its own resolution purpose sought to be accomplished thereby, in order to construe the whole as to make
providing that it will not consider any election protest that was not submitted on or before the words consonant to that reason and calculated to effect that purpose
December 9, 1935.Citing among others the earlier resolution of the National Assembly, the
petitioner sought the dismissal of respondent’s protest. The Electoral Commission however PHILIPPINE BASELINE LAW - Magallona vs. Ermita Case
denied his motion. Ynsua contended that the Electoral Commission act without or in excess
In Magallona vs Ermita, a group of law students along with their professors and a party-list
of its jurisdiction in taking cognizance of the protest filed against the election of the petitioner
representative, in their respective capacities as citizens, taxpayers and legislators,
notwithstanding the previous confirmation of such election by resolution of the National
challenged the constitutionality of the R.A. No. 9522, (the Philippine Baseline Law) for
Assembly
reducing the Philippine state’s sovereign power by reducing the Philippines’ maritime
The court ruled that the Electoral Commission did not act without or in excess of its territory, undermining Philippine sovereignty and national territory and contravening the
jurisdiction in taking cognizance of the protest filed against the election of the petitioner country’s nuclear-free policy.
notwithstanding the previous confirmation of such election by resolution of the National
Assembly. The court ruled that Whether referred to as Philippine "internal waters" under Article I of
the Constitution or as "archipelagic waters" under UNCLOS III, the Philippines exercises
The Electoral Commission acted within the legitimate exercise of its sovereignty over the body of water lying landward of the baselines, including the air space
CONSTITUTIONAL PREROGATIVE in assuming to take cognizance of the protest over it and the submarine areas underneath. RA 9522 simply seeks to conform to our
filed by the respondent Ynsua against the election of the petitioner Angara, and that international agreement on the setting of baselines and provides nothing about the
the earlier resolution of the National Assembly cannot in any manner toll the time for designation of archipelagic sea-lane passage or the regulation of innocent passage within
filing election protests against members of the National Assembly, nor prevent the our waters
filing of a protest within such time as the rules of the Electoral Commission might
prescribe.
CONSTI REVIEWER 2

The enactment of UNCLOS III compliant baselines law for the Philippine archipelago SUIT AGAINST THE REPUBLIC OF THE PHILIPPINES -Republic vs. Villasor Case
and adjacent areas, as embodied in RA 9522, allows an internationally-recognized
delimitation of the breadth of the Philippines’ maritime zones and continental shelf. - The Republic of the Philippines in this case is a certiorari and prohibition proceeding
RA 9522 is therefore a most vital step on the part of the Philippines in safeguarding its challenging the validity of an order issued by respondent Judge Guillermo P. Villasor, which
maritime zones, consistent with the Constitution and our national interest. declares a decision final and executory and of an alias writ of execution directed against the
The breadth of the territorial sea, the contiguous zone, the exclusive economic zone funds of the Armed Forces of the Philippines subsequently issued in pursuance thereof, the
and the continental shelf shall be measured from archipelagic baselines drawn in alleged ground being excess of jurisdiction, or at the very least, grave abuse of discretion.
accordance with article 47. The Republic of the Philippines is entitled to the writs prayed for. Respondent Judge ought
not to have acted thus. The order thus impugned and the alias writ of execution must be
nullified.
DOCTRINE OF PARENS PATRIAE
Cabañas vs. Pilapil, GR L-25843, July 25, 1974 - The State cannot be sued without its consent. It is a fundamental postulate of
constitutionalism flowing from the juristic concept of sovereignty that the state as well as its
This is an appeal from a question of law from a lower court decision regarding the proceeds government is immune from suit unless it gives its consent. It is readily understandable why
of an insurance policy issued on the life of a deceased father of a minor beneficiary. The it must be so. In the classic formulation of Holmes: “A sovereign is exempt from suit, not
dispute centers as to who of the mother of the child and the brother of the deceased should because of any formal conception or obsolete theory, but on the logical and practical ground
be entitled to act as trustee thereof. that there can be no legal right as against the authority that makes the law on which the right
depends.”
The deceased, Florentino Pilapil, insured himself and instituted as beneficiary, his child,
Millian Pilapil, with his brother to act as trustee during her minority. Upon his death, the - Judgment against the State cannot be enforced by execution. The universal rule that
proceeds were paid to Francisco Pilapil, Florentino’s brother. Hence a complaint by the where the State gives its consent to be sued by private parties either by general or
mother, Melchora Cabañas, with whom the child is living, seeking the delivery of such sum. special law, it may limit claimant’s action “only up to the completion of proceedings
anterior to the state of execution” and that the power of the Courts ends when the
The brother raised an appeal on question of law as the trial court decided in favor of the judgment is rendered, since government funds and properties may not be seized
mother. The Court decided on the affirmative to the decision of the lower court, stating that under writs of execution or garnishment to satisfy such judgments, is based on
“It is very clear, therefore, considering the above, that unless the applicability of Articles 320 obvious considerations of public policy. The functions and public services rendered
and 321 of the Civil Code cited by the trial court can be disputed, the decision must stand. by the State cannot be allowed to be paralyzed or disrupted by the diversion of public
There is no ambiguity in the language employed. The words are rather clear. Their meaning funds from their legitimate and specific objects, as appropriated by law.
is unequivocal. Time and time again, this Court has left no doubt that where codal or statutory
norms are cast in categorical language, the task before it is not one of interpretation but of SUIT AGAINST FOREIGN STATE – Minucher vs. CA; G.R. No. 142396 February 11, 2003
application. So it must be in this case. So, it was in the appealed decision.”
Minucher filed for damages against Arthur Scalzo for trumped-up charges of drug trafficking.
The judiciary, as an agency of the State acting as parens patriae, is called upon Scalzo counterclaimed that he acted within his official capacity as an agent of the Drug
whenever a pending suit of litigation affects one who is a minor to accord priority to Enforcement Administration of the United States Department of Justice and filed a motion to
his best interest. ... What is more, there is this constitutional provision vitalizing this dismiss on the account of his diplomatic immunity.
concept. It reads: “The State shall strengthen the family as a basic social institution.”
If, as the Constitution so wisely dictates, it is the family as a unit that has to be Minucher’s petition is DENIED. A foreign agent, operating within a territory, can be cloaked
strengthened, it does not admit of doubt that even if a stronger case were presented with immunity from suit but only as long as it can be established that he is acting within the
for the uncle, still deference to a constitutional mandate would have led the lower court directives of the sending state. The consent of the host state, with respect to the foreign
to decide as it did agent’s directives, is an indispensable requirement of basic courtesy between the two
sovereigns. Consent or imprimatur of the Philippine government can be gleaned from the
official communications between the two countries’ government agencies, certifications from
the PH-DFA and US Embassy, as well as the participation of members of the Philippine
Narcotics Command.
CONSTI REVIEWER 3

International Law; Vienna Convention on Diplomatic Relations; Diplomatic Missions; INCORPORATED – TEST OF SUABILITY IS FOUND IN ITS CHARTER (IT IS SUABLE IF
Diplomatic Immunity; Suing a representative of a state is believed to be, in effect, suing ITS CHARTER SAYS SO, REGARDLESS OF THE FUNCTIONS IT IS PERFORMING)
the state itself—the proscription is not accorded for the benefit of an individual but for
the State, in whose service he is, under the maxim—par in parem, non habet imperium. Olizon vs. Central Bank of the Philippines
- If the acts giving rise to a suit are those of a foreign government done by its foreign agent,  This case is about the defendant Central Bank on various dates collected from the
although not necessarily a diplomatic personage, but acting in his official capacity, the plaintiff Olizon amounts for the payment of Special Excise Tax on Foreign Exchange pursuant
complaint could be barred by the immunity of the foreign sovereign from suit without its to its Monetary Board Resolution No. 286. On March 10, 1958, Olizon then requested the
consent defendant Bank to refund to him the amounts he paid plus the sum of P463.54 but the Central
SOVEREIGN EQUALITY OF STATES (Suits against foreign States)-ALMEDA V. SYQUIA Bank refused to accede to these requests and concedes the illegality of the resolution under
which it made the levy.
The Syquia brothers (Pedro, Gonzalo, Leopoldo) are the undivided joint owners of three
apartment buildings in Manila. Wherein the plaintiffs entered into a lease contract with the  The court ruled that the suit is brought against the Central Bank of the Philippines,
United States of America and sued the USA for non-compliance with the contract. an entity authorized by its charter to sue and be sued. The consent of the State to thus be
sued, therefore, has been given. As said in the case of Central Azucarera San Pedro vs.
The Municipal Trial Court dismissed the case on the grounds of that under the well settled Central Bank, 104 Phil., 598, in suits for refund, "being a corporation that may sue and be
rule of International Law, a foreign government like the United States Government cannot be sued, the Central Bank is the proper party defendant pursuant to Section 5 of Republic Act
sued in the courts of another state without its consent. It was appealed to CFI Manila wherein No. 601, which provides that "the refund of taxes pursuant to sections two and three of this
the court also affirmed the decision of the MTC. It was appealed to the Supreme Court which Act shall be made by the Central Bank of the Philippines."
dismissed the said case and affirmed the decision of the MTC and CFI Manila.
 A suit brought against the Central Bank, an entity authorized by its charter to sue
A foreign government like the United States Government cannot be sued in the courts and be sued, is not a suit against the State without its consent.
of another state without its consent.
SUIT AGAINST THE UN AND OTHER SPECIALIZED AGENCIES – LASCO VS.
UNRFNRE
MUST NOT PERPETRATE AN INJUSTICE (STATE IMMUNITY FROM SUIT) - Amigable Petitioners were dismissed from employment by United Nations Revolving Fund for Natural
vs. Cuenca Case Resources Exploration (UNRFNRE), a special fund and subsidiary organ of UN involved in
a joint project of the Philippine Government and the United Nations for exploration work in
In Amigable vs. Cuenca, Victoria Amigable’s property were used by the government for the Dinagat Island. Aggrieved, the petitioners filed a complaint against UNFRNRE for illegal
construction of avenues (6,167 square meters) without any expropriation or negotiated sale. dismissal and damages. The case was dismissed by Labor Arbiter and such dismissal was
After her request for payment was denied by the Auditor General, her complaint for recovery affirmed by NLRC.
of ownership was dismissed for lack of jurisdiction on the ground that the government cannot
be sued without its consent. The court ruled that immunity is necessary to assure unimpeded performance of their
functions. The grant of privileges and immunities to international organizations, its officials
The court rule that where the government takes away property from a private landowner for and functionaries is to secure them legal and practical independence. The mission of
public use without going through the legal process of expropriation or negotiated sale, the UNRFNRE is to help improve the quality of the people, including the petitioners, and not to
aggrieved party may properly maintain a suit against the government without thereby exploit our resources.
violating the doctrine of governmental immunity from suit without its consent.
Sec. 4, Art IV of the 1946 Convention on the Privileges and Immunities of the
The doctrine of governmental immunity from suit cannot serve as an instrument for Specialized Agencies of the United Nations: Specialized agencies xxx shall enjoy
perpetrating an injustice on a citizen. immunity from every form of legal process except in particular case that they have
waived their immunity.
CONSTI REVIEWER 4

SUABILITY VS. EXECUTION – COMMISSIONER OF PUBLIC HIGHWAYS VS. SAN works. Such act, although not directly a governmental function, is a necessary incident in
DIEGO their course.
- Tomas Hashim, the Judicial Administrator of N.T. Hashim filed a claim against the The suit disguised as one to compel the Auditors to approve the vouchers for
Commissioner of Public Highways for compensation on the land transferred to the payment, is a suit against the State, which cannot prosper or be entertained by the
government for the construction of EDSA. The Court ordered a writ of execution and Court except with the consent of the State.
garnishment for the release of the amount as agreed under the parties’ compromise
agreement. Subsequently, PNB transferred the amount to the respondents without the INCORPORATED/UNINCORPORATED-IF GOVERNMENTAL FUNCTION, CANNOT BE
consent of the petitioners. SUED, IF NOT GOVERNMENTAL, FUNCTION CAN BE SUED- Bureau of Printing v.
Bureau of Printing Employees Association)
- The Supreme Court held that respondent court’s orders are null and void on the ground that
government funds are not subject to execution or garnishment. Doctrine of immunity does This case is about the issue between the Bureau of Printing (an UNINCORPORATED
not apply to expropriation proceedings. company) administered by the executive secretary, office of the President and is charged
with the printing and binding including work incidental and Bureau of Printing employees to
Through R.A. 3083, the government of the Philippines consents and submits to be which the employees of the Bureau of printing filed a case against the company due to
sued upon any money claim involving liability arising from contracts, express or alleged unfair labor practice for receiving outside jobs and that many of its employees are
implied, which could serve as basis of civil action between private parties. paid overtime work on regular working days and holidays.
SUIT AGAINST GOVERNMENT WHEN ENTERING INTO CONTRACT/SUABILITY VS. The court ruled that as an office of the government without any corporate or juridical
LIABILITY – Merritt vs. Government; G.R. No. L-11154 March 21, 1916 personality, the Bureau of Printing cannot be sued, because its functions are still considered
a governmental function, and the rule is settled that the government cannot be sued without
Merritt collided with a General Hospital ambulance and was severely injured which resulted its consent.
as a serious detriment to his work and subsequently, life. An act was authorized allowing
Merritt to bring suit against the Government of the Philippines Islands. Merritt used this act The Bureau of Printing is primarily a service bureau and is not engaged in its business
as evidence for the admittance of liability by the Philippine Government in favor of Merritt. or occupation for pecuniary profit. Although it receives outside jobs and many of its
employees are paid for overtime work on regular working days and holidays, these
The waiving of immunity from suit does not give rise to a liability not previously existing. The facts do not justify the conclusion that its functions are “exclusively proprietary in
Government of the Philippine Islands is only liable for the negligence of its officers, agents, nature”.
and employees when they are acting in a special capacity.
Special Statutes; Consent of the State to be Sued; Construction - An act permitting a ULTIMATE LIABILITY WILL FALL ON THE GOVERNMENT – Calub and Valencia vs CA,
suit against the state gives rise to no liability not previously existing unless it is clearly Brief Summary: The case is about DENR officers Felipe Calub and Ricardo Valencia
expressed in the act. assailing that the complaint for the recovery of possession of the two impounded vehicles,
NO SUIT WITHOUT CONSENT WHEN THE ACT WAS A NECESSARY INCIDENT TO ITS with an application for Replevin, is a suit against the state.
GOVERNMENTAL FUNCTION- Sayson vs Singson Case Ruling: The Court ruled that the acts in question are clearly official in nature. In implementing
This case is about Singson, owner of Singkier Motor Service, suing the Highway Auditor, and enforcing Sections 78-A and 89 of the Forestry Code through the seizure carried out,
Sayson, for withholding 20% of the approved payments, for reasons of overpricing, as per petitioners were performing their duties and functions as officers of the DENR, and did so
the service contract between them over which the said contract is for the parts needed for within the limits of their authority. There was no malice nor bad faith on their part. Hence, a
the reparation of D8 Bulldozer. suit against the petitioners who represent the DENR is a suit against the state which cannot
prosper without the its consent.
The court ruled that Sayson, in behalf of the Public works and communication, cannot be
sued since such act constitutes a suit against the government which is prohibited. The spare Jurisprudence: A suit against a public officer for his official acts is, in effect, a suit
parts ordered are for the reparation of an instrument used primarily in public construction against the State if its purpose is to hold the State ultimately liable.
CONSTI REVIEWER 5

EXCEPTION – NO SUIT WITHOUT CONSENT WHEN THE ACT WAS A NECESSARY into a contract. That the correct test for the application of State immunity is not the
INCIDENT TO ITS GOVERNMENT FUNCTION – Mobil Philippines Exploration v Customs conclusion of a contract by a State but the legal nature of the act is shown in Syquia vs.
Arrastre Service Case Lopez. In Syquia, the United States concluded contracts with private individuals but the
contracts notwithstanding the States was not deemed to have given or waived its
Four cases of rotary drill parts were shipped from abroad, consigned to Mobil Philippines consent to be sued for the reason that the contracts were for jure imperii and not for
Exploration, Inc. The shipment was discharged to the custody of the Customs Arrastre jure gestionis.
Service, the unit of the Bureau of Customs then handling arrastre operations therein. The
Customs Arrastre Service later delivered to the broker of the consignee three cases only of The traditional rule of State immunity exempts a state from being sued in the courts
the shipment. Mobil Philippines Exploration, Inc filed suit in the Court of First Instance of of another state without its consent or waiver. This rule is a necessary consequence
Manila against the Customs Arrastre Service and the Bureau of Customs to recover the value of the principles of independence and equality of states. However, the rules of
of the undelivered case plus other damages. international law are not petrified; they are constantly developing and evolving. And
because the activities of states have multiplied, it has been necessary to distinguish
The Court ruled that although said arrastre function may be deemed proprietary, it is a them — between sovereign and governmental acts and private, commercial and
necessary incident of the primary and governmental function of the Bureau of Customs, so proprietary acts. The result is that state immunity now extends only to sovereign and
that engaging in the same does not necessarily render said Bureau liable to suit. For governmental acts.
otherwise, it could not perform its governmental function without necessarily exposing itself
to suit. Sovereign immunity, granted as to the end, should not be denied as to the necessary The restrictive application of state immunity is proper only when the proceedings arise
means to that end. out of commercial transactions of the foreign sovereign, its commercial activities or
economic affairs. A state may be said to have descended to the level of an individual
State cannot be sued without its consent – Regardless of the merits of the claim and can thus be deemed to have tacitly given its consent to be sued only when it
against it, the State, for obvious reason of public policy, cannot be sued without its enters into business contracts. It does not apply where the contract relates the
consent exercise of its sovereign function. In this case, the projects are an integral part of the
STATE IMMUNITY WHEN IT ENTERS INTO CONTRACT naval base which is devoted to the defense of both the US and the Philippines,
U.S. vs. Ruiz, GR L-35645, May 22, 1985 indisputably a function of the government of the highest order; they are not utilized
for nor dedicated to commercial or business purposes.
The United States of America had a naval base in Subic, Zambales. The base was one
of those provided in the Military Bases Agreement between the Philippines and the US.
The US made an invitation for the submission of bids for the repair of wharves in said WHEN IT ENTERS INTO CONTRACT- US. vs. Guinto Case
base. Private respondent Eligio de Guzman & Co., Inc. responded to the invitation and - The private respondents are suing several officers of the U.S. Air Force stationed in Clark
submitted bids. Subsequent thereto, the company received from the United States two Air Base in connection with the bidding conducted by them for contracts for barbering
telegrams requesting it to confirm its price proposals and for the name of its bonding services in the said base, which was won by Dizon. The private respondents object that,
company. The company complied with the requests. Dizon had made a bid for four facilities, which was not included in the invitation to bid.
The company later received a letter signed by Wilham I. Collins, one of the petitioners, - State Immunity from Suit; When the government enters into a contract, it is deemed
stating that the company did not qualify to receive an award for the projects because of to have descended to the level of the other contracting party, and divested of its
its previous unsatisfactory performance rating on a repair contract for the sea wall at sovereign immunity from suit with its implied consent. The general law waiving the
the boat landings of the U.S. Naval Station in Subic Bay. The letter further said that the immunity of the state from suit is found in Act No. 3083, under which the Philippine
projects had been awarded to third parties. government “consents and submits to be sued upon any moneyed claim involving liability
Respondent alleges that it won in the bidding conducted by the US for the construction arising from contract, express or implied, which could serve as a basis of civil action between
of wharves in said base that was wrongly awarded to another group. For this reason, a private parties.” In Merritt v. Government of the Philippine Islands, a special law was passed
suit for specific performance was filed by him against the US. to enable a person to sue the government for an alleged tort. When the government enters
into a contract, it is deemed to have descended to the level of the other contracting party and
The Court ruled that the US is has not waived its immunity from suit despite entering divested of its sovereign immunity from suit with its implied consent. Waiver is also implied
CONSTI REVIEWER 6

when the government files a complaint, thus opening itself to a counterclaim. The above rules regard to the Compromise Agreement made between the PCGG and private respondent,
are subject to qualification. Express consent is effected only by the will of the legislature Roberto S. Benedicto.
through the medium of a duly enacted statute. We have held that not all contracts entered
into by the government will operate as a waiver of its non-suability; distinction must be made The Supreme Court dismissed the petition for certiorari and held that the Sandiganbayan did
between its sovereign and proprietary acts. As for the filing of a complaint by the government, not commit any error as to finding PCGG liable for the loss of the said shares of respondent,
suability will result only where the government is claiming affirmative relief from the Benedicto. It also stated that the Immunity against State could not be invoked due to its
defendant. Compromise Agreement done with the respondent, Benedicto.

- Rule on waiver, not applicable when the contract entered into involves its sovereign When the State enters into contract, through its officers or agents, in furtherance of a
or governmental capacity. There is no question that the United States of America, like legitimate aim and purpose and pursuant to constitutional legislative authority,
any other state, will be deemed to have impliedly waived its non-suability if it has whereby mutual or reciprocal benefits accrue and rights and obligations arise
entered into a contract in its proprietary or private capacity. It is only when the contract therefrom, the State may be sued even without its express consent, precisely because
involves its sovereign or governmental capacity that no such waiver may be implied. by entering into a contract the sovereign descends to the level of the citizen.
This was our ruling in United States of America v. Ruiz, where the transaction in
question dealt with the improvement of the wharves in the naval installation at Subic
Bay. As this was a clearly governmental function, we held that the contract did not NON – DELEGABILITY OF LEGISLATIVE POWER – BELGICA VS. OCHOA
operate to divest the United States of its sovereign immunity from suit.
The pork barrel system has been around in the Philippines since about 1922. Pork Barrel is
commonly known as the lump-sum, discretionary funds of the members of the Congress. It
ULTIMATE LIABILITY WILL FALL ON THE OFFICER – RUIZ VS. CABAHUG went through several legal designations from “Congressional Pork Barrel” to the latest
“Priority Development Assistance Fund” or PDAF. The allocation for the pork barrel is
- National Defense Secretary Cabahug accepted the bid of the Allied Technologists, Inc., to integrated in the annual General Appropriations Act (GAA).
furnish architectural and engineering services at a price of P302,700.When the officials of the
Department of National Defense paid the Allied Technologists the contract price for the The court observed that it violates the principle of non – delegability since individual
architectural engineering services, they retained 1 of the sum due. This action then deprived legislators, in this case, are allowed to exercise the power of appropriation. The legislative
Mr. Ruiz and Mr. Herrera monetary value of their professional services and damaged their power cannot be delegated by Congress for it cannot delegate further that which was
professional prestige and standing. delegated to it by the Constitution.
- The court ruled that the suit should be answered not by the government but to its officials Sec. 29 (1), Art. VI of the 1987 Constitution clearly states that the power of
to compel them to act in accordance with the rights to be established by the contending appropriation must be exercised through legislation.
architects, or to prevent them from making payment and recognition until the contending
architects have established their respective rights and interest in the funds retained and in POTESTAS DELEGATA NON POTEST DELEGARE – Jaworski vs. PAGCOR
the credit for the work done. PAGCOR issued and authorized SAGE to operate Sports betting stations in PAGCOR’s
- The Government is not liable for the actions done by their public officials beyond Casino location and Internet Gaming facilities. Sen. Jaworski filed for the nullification of the
their official duties. And if the officials acted beyond their authority to perform their grant of authority given by PAGCOR to SAGE. Jaworski contends that PAGCOR is not
official duties, a suit is filed not against the State but against the Officials. authorized to authorize other corporations under its franchise.

IMPLIED WAIVER OF IMMUNITY (Immunity of State against suit)-REPUBLIC V. The Grant of Authority by PAGCOR in favor of SAGE is NULL and VOID. PAGCOR cannot
SANDIGANBAYAN delegate its power in view of the legal principle of delegata potestas delegare non potest,
inasmuch as there is nothing in the charter to show that it has been expressly authorized to
The respondent Sandiganbayan holds PCGG at fault for not paying the membership dues on do so.
the 227 sequestered NOGCCI shares of stock, a failing which eventually led to the
foreclosure sale thereof thus being unable to comply with the Sandiganbayan’s order with Constitutional Law; Government; Powers; Legislative Department; Legislative
Franchise; A legislative franchise is a special privilege granted by the state to
CONSTI REVIEWER 7

corporations. - It is a privilege of public concern which cannot be exercised at will and - With regards to the principle of Separation of Powers, each department of the government
pleasure, but should be reserved for public control and administration, either by the has exclusive cognizance of the matters within its jurisdiction, and is supreme within its own
government directly, or by public agents, under such conditions and regulations as the sphere. In cases of conflict, the judicial department is the only constitutional organ
government may impose on them in the interest of the public. which can be called upon to determine the proper allocation of powers between the
several departments and among the integral or constituent units thereof.

SEPARATION OF POWERS- BELGICA V. OCHOA Case DOCTRINE OF CHECKS AND BALANCE – Belgica vs Ochoa, GR 208566

This case is about the issue on “PORK BARREL” the petitioners herein seek to declare the Brief Summary: The case is about the assailing the constitutionality of the 2013 Priority
“Pork Barrel System” to be considered unconstitutional for constituting grave abuse of Development Assistance Fund (PDAF) and the Pork Barrel System.
discretion as well as the Executive’s lump sum, discretionary finds, such as the Malampaya
Ruling: The Court ruled that since the intermediate appropriations are made by legislators
Funds and the Presidential Social Fund. They also contended that the 2013 PDAF Article
only after the General Appropriations Act (GAA) is passed, and hence outside of the law, it
and all other Congressional Pork Barrel Laws similar thereto are violation of the principle of
necessarily means that the actual items of PDAF appropriation would not have been written
Separation of Powers.
in the General Appropriations Bill and thus effectuated without veto consideration. This kind
The court ruled that the “Pork Barrel System” is a violation of the principle of Separation of of lump sum/post enactment legislative identification budgeting system fosters the creation
Powers because it was mentioned that the legislature/congress in the implementation of the of a budget within a budget which subverts the prescribed procedure of presentment and
pork barrel system has activity in post-enactment measures which govern the project consequently impairs the President’s power of item veto.
identification, fund release and fund realignment are clearly not related to functions of
Jurisprudence: A prime example of a constitutional check and balance would be the
congressional oversight which allow legislators to intervene and assume duties that properly
President’s power to veto an item written into an appropriation, revenue or tariff bill
belong to the budget execution.
submitted to him by Congress for approval through a process known as “bill
The legislature has no authority to execute or construe the law, the executive has no presentment”. The President’s item-veto power is found in Section 27(2), Article VI of
authority to make or construe law, and the judiciary has no power to make or execute the 1987 Constitution.
law. From the moment the law becomes effective, any provision of law that empowers
WHICH DEPARTMENT ENSURES THE CONSTITUTIONAL DISTRIBUTION OF
Congress or any of its members to play any role in the implementation of enforcement
POWERS? DOES IT MEAN SUCH DEPARTMENT IS SUPERIOR TO THE OTHER
of the law violates the principle of SEPARATION OF POWERS. (ABAKADA CASE)
DEPARTMENTS?
Abakada Guro Party List vs. Purisima
OURS IS A GOVERNMENT OF LAWS AND NOT MEN
 This case is about petitioners assailing the creation of a congressional oversight
Does it mean such department is superior to the other departments? – Angara vs.
committee on the ground that it violates the doctrine of separation of powers. While the
Electoral Commission
legislative function is deemed accomplished and completed upon the enactment and
- Petitioner Angara was proclaimed as winner of the position as the member of the National approval of the law, the creation of the congressional oversight committee permits
Assembly, days after, the National Assembly had passed a resolution regarding those who legislative participation in the implementation and enforcement of the law.
have not been subject of an election protest. However, respondent Ynsua filed an election
 The court declined jurisdiction on this case. The Joint Congressional Oversight
protest against the petitioner before the Electoral Commission. The Petitioner sought the
Committee in RA 9355 was created for the purpose of approving the implementing rules
dismissal of the respondent’s protest but was denied by the Electoral Commission.
and regulations (IRR) formulated by the DOF, DBM, NEDA, BIR, BOC, and CSC. On
- The Electoral Commission did not act without or in excess of its jurisdiction. The nature of May 22, 2006, it approved the said IRR. From then on, it became functus officio and
the controversy shows the necessity of a final constitutional arbiter to determine the conflict ceased to exist. Hence, the issue of its alleged encroachment on the executive function
of authority between two agencies created by the Constitution. Upon principle, reason and of implementing and enforcing the law may be considered moot and academic.
authority, the Supreme Court has jurisdiction over the Electoral Commission and the subject
matter of the present controversy.
CONSTI REVIEWER 8

 Based from the ruling under Macalintal v. Commission on Elections, it is clear that  Truth be told, the declared unconstitutionality of the aforementioned phrase is but an
congressional oversight is not unconstitutional per se, meaning, it neither necessarily assurance that the Malampaya Funds would be used — as it should be used — only in
constitutes an encroachment on the executive power to implement laws or undermines accordance with the avowed purpose and intention of PD 910.
the constitutional separation of powers. Rather, it is integral to the checks and balances
inherent in a democratic system of government. It may in fact even enhance the TRULY POLITICAL QUESTION VS. NOT TRULY POLITICAL QUESTION
separation of powers as it prevents the over-accumulation of power in the executive Sufficiency Standard Test – Gerochi vs. Dept. of Energy
branch.
R.A. 9136 or the Electric Power Industry Reform Act of 2001, which sought to impose a
SUFFICIENCY STANDARD TEST universal charge on all end-users of electricity for the purpose of funding NAPOCOR’s
Belgica vs. Ochoa (“undue delegation” with regard to presidential pork barrel) projects was enacted and took effect in 2001. The petitioners contest the said act’s
constitutionality, stating that the imposition of the universal charge on all end-users is
 One of the facts of this case is the History of the Presidential Pork Barrel in The oppressive and confiscatory and amounts to taxation without representation for not giving the
Philippines by which the Malampaya Funds was created as a special fund under consumers a chance to be heard and be represented.
Section 8 of Presidential Decree No. (PD) 910, issued by then President Ferdinand E.
Marcos (Marcos) on March 22, 1976. In enacting the said law, Marcos recognized the Reiterated in the National Association of Electricity Consumers for Reforms vs. Energy
need to set up a special fund to help intensify, strengthen, and consolidate government Regulatory Commission, where the court held that the ERC, as regulator, should have
efforts relating to the exploration, exploitation, and development of indigenous energy sufficient powers to respond in real time to changes wrought by multifarious factors affecting
resources vital to economic growth. Due to the energy-related activities of the public utilities. The court held that there is no undue delegation of legislative power to the
government in the Malampaya natural gas field in Palawan, or the "Malampaya Deep ERC.
Water Gas-to-Power Project", the special fund created under PD 910 has been All that is required for the valid exercise of the power of subordinate legislation is that the
currently labeled as Malampaya Funds. regulation be germane to the objects and purposes of the law and that the regulation be not
 The court agrees with petitioners that the phrase “and for such other purposes as may in contradiction to, but in conformity with, the standards prescribed by law. And these
be hereafter directed by the President” under Section 8 of PD 910 constitutes an undue requirements are the completeness and the sufficient standard tests.
delegation of legislative power insofar as it does not lay down a sufficient standard to TRULY POLITICAL QUESTION V NOT TRULY POLITICAL QUESTION
adequately determine the limits of the President’s authority with respect to the purpose
for which the Malampaya Funds may be used. As it reads, the said phrase gives the This issue is a non-justiciable political question which is beyond the scope of the judicial
President wide latitude to use the Malampaya Funds for any other purpose he may power of the Supreme Court under Section 1, Article VIII of the Constitution.
direct and, in effect, allows him to unilaterally appropriate public funds beyond the
purview of the law. That the subject phrase may be confined only to “energy resource 1. Any discussion of this issue would require the Court to make a determination of what
development and exploitation programs and projects of the government” under the constitutes an impeachable offense. Such a determination is a purely political question
principle of ejusdem generis, meaning that the general word or phrase is to be which the Constitution has left to the sound discretion of the legislation. Such an intent is
construed to include — or be restricted to — things akin to, resembling, or of the same clear from the deliberations of the Constitutional Commission.
kind or class as those specifically mentioned, is belied by three (3) reasons: first, the
phrase “energy resource development and exploitation programs and projects of the 2. Courts will not touch the issue of constitutionality unless it is truly unavoidable and is the
government” states a singular and general class and hence, cannot be treated as a very lis mota or crux of the controversy.
statutory reference of specific things from which the general phrase “for such other Truly political questions are beyond judicial review while courts can review questions
purposes” may be limited; second, the said phrase also exhausts the class it which are not truly political in nature. —Truly political questions are thus beyond
represents, namely energy development programs of the government; and, third, the judicial review, the reason for respect of the doctrine of separation of powers to be
Executive department has, in fact, used the Malampaya Funds for non-energy related maintained. On the other hand, by virtue of Section 1, Article VIII of the Constitution,
purposes under the subject phrase, thereby contradicting respondents’ own position courts can review questions which are not truly political in nature.
that it is limited only to “energy resource development and exploitation programs and
projects of the government.”
CONSTI REVIEWER 9

POLITICAL QUESTIONS VS NOT TRULY POLITICAL QUESTIONS- Tanada vs Angara reasons for the enforcement of the doctrine. In the case of the foreign state sought to be
impleaded in the local jurisdiction, the added inhibition is expressed in the maxim par
This is a case petition by Sen. Wigberto Tanada, together with other lawmakers, taxpayers, in parem, non habet imperium. All states are sovereign equals and cannot assert
and various NGO’s to nullify the Philippine ratification of the World Trade Organization (WTO) jurisdiction over one another. A contrary disposition would, in the language of a
Agreement. The contention relative to the topic is that it is in conflict with the constitution, celebrated case, “unduly vex the peace of nations.” [De Haber v. Queen of Portugal, 17
since the said Agreement is an assault on the sovereign powers of the Legislative because Q. B. 171] While the doctrine appears to prohibit only suits against the state without its
it meant that Congress could not pass legislation that would be good for national interest and consent, it is also applicable to complaints filed against officials of the state for acts
general welfare if such legislation would not conform to the WTO Agreement over which allegedly performed by them in the discharge of their duties. The rule is that if the
constitutes a Justiciable controversy. judgment against such officials will require the state itself to perform an affirmative act to
The court ruled that this case falls under the not truly political question over which the judiciary satisfy the same, such as the appropriation of the amount needed to pay the damages
has jurisdiction. Judicial power includes the duty of the courts of justice to settle actual awarded against them, the suit must be regarded as against the state itself although it has
controversies involving rights which are legally demandable and enforceable, and to not been formally impleaded. [Garcia v. Chief of Staff, 16 SCRA 120] In such a situation, the
determine whether or not there has been a grave abuse of discretion amounting to lack or state may move to dismiss the complaint on the ground that it has been filed without its
excess of jurisdiction on the part of any branch or instrumentality of the government. consent.

Constitutional Law; Judicial Review; Separation of Powers;Where an action of the TRULY POLITICAL QUESTIONS VS. NOT TRULY POLITICAL QUESTIONS - Defensor
legislative branch is seriously alleged to have infringed the Constitution, it becomes vs. Guingona Case
not only the right but in fact the duty of the judiciary to settle the dispute.\ In Defensor vs. Guingona, Senators Miriam Defensor Santiago and Francisco S. Tatad
TRULY POLITICAL QUESTIONS VS NOT TRULY POLITICAL-Arigo vs. Swift Case sought the ouster of Senator Teofisto T. Guingona, Jr. as minority leader of the Senate and
the declaration of Senator Tatad as the rightful minority leader on the grounds that those who
This case is a petition for the issuance of a Writ of Kalikasan with prayer for the issuance of voted for the winning Senate President shall compose the “majority” on those who did not,
a Temporary Environmental Protection Order (TEPO) under Rule 7 of A.M. No. 09-6-8-SC, the “minority”. Sen. Guingona, having voted for Sen. Marcelo B. Fernan, thus is not of the
otherwise known as the Rules of Procedure for Environmental Cases (Rules), involving “minority; alleging in the main that Senator Guingona had been usurping, unlawfully holding
violations of environmental laws and regulations in relation to the grounding of the US military and exercising the position of Senate minority leader, a position that, according to them,
ship USS Guardian over the Tubbataha Reefs. rightfully belonged to Sen. Tatad
The immunity of the State from suit, known also as the doctrine of sovereign immunity or The court rule that jurisdiction over the subject matter of a case is determined by the
non-suability of the State, is expressly provided in Article XVI of the 1987 Constitution which allegations of the complaint or petition, regardless of whether the plaintiff or petitioner is
states: “Section 3. The State may not be sued without its consent.” In United States of entitled to the relief asserted; and that it is well within the power and jurisdiction of the Court
America v. Judge Guinto, we discussed the principle of state immunity from suit, as follows: to inquire whether indeed the Senate or its officials committed a violation of the Constitution
The rule that a state may not be sued without its consent, now expressed in Article XVI, or gravely abused their discretion in the exercise of their functions and prerogative.
Section 3, of the 1987 Constitution, is one of the generally accepted principles of international
law that we have adopted as part of the law of our land under Article II, Section 2. x x x. Even While no provision of the Constitution or the laws or the rules and even the practice
without such affirmation, we would still be bound by the generally accepted principles of of the Senate was violated, and while the judiciary is without power to decide matters
international law under the doctrine of incorporation. Under this doctrine, as accepted by the over which full discretionary authority has been lodged in the legislative department,
majority of states, such principles are deemed incorporated in the law of every civilized state the Supreme Court may still inquire whether an act of Congress or its officials has
as a condition and consequence of its membership in the society of nations. Upon its been made with grave abuse of discretion (not truly political).
admission to such society, the state is automatically obligated to comply with these principles SELF-EXECUTING VS. NON-SELF-EXECUTING – Imbong vs. Ochoa
in its relations with other states.
Petitioners assail the Constitutionality of the proposed RH Law on several grounds.
As applied to the local state, the doctrine of state immunity is based on the justification given Petitioners contend that RA 10354 violate Art. II, Secs. 12 and 15.
by Justice Holmes that “there can be no legal right against the authority which makes the law
on which the right depends.”[Kawanakoa v. Polybank, 205 U.S. 349] There are other practical
CONSTI REVIEWER 10

General rule is that Constitutional provisions are Self-Executing unless it is expressly cases where the Chief executive is required by the Constutiion or the law to act in cases
provided that legislature is necessary. The Principles and State Policies enumerated in Article where the Chief executive is demand that he act personally.
II of the Constitution are NOT SELF-EXECUTING PROVISIONS. RA 10354 NOT
UNCONSTITUTIONAL as it takes great cares to comply with the principles cited from Art. II, The fact, however that the power of suspension is expressy granted by section 2188
particularly the right to life/sanctity of family life and right to health. of the Administrative Code to the provincial governor does not mean that the grant is
necessarily exclusive and precludes the Secretary of Interior from exercising a similar
Reproductive Health Law; Contraceptives power.
SELF-EXECUTING/SELF-RELIANT AND INDEPENDENT ECONOMY - Tanada v. Angara
SELF EXECUTING AND NON-SELF EXECUTING - GAMBOA V. TEVES Rizalino Navarro, then secretary of Department of Trade and Industry representing the
Government of the Philippines signed in Marakesh Morocco the Final Act embodying the
The issue started when petitioner Gamboa questioned the indirect sale of shares to First results of the Uruguay Round Multilateral Negotiations regarding the WTO agreement.
Pacific’s common shareholdings in PLDT increased from 30.7 percent to 37 percent, thereby Tanada, as member of the Philippine Senate filed the case for certiorari contending that this
increasing the total common shareholdings of foreigners in PLDT to about 81.47%. The will be detrimental to the growth of our National Economy and against to the “Filipino First”
petitioner contends that it violates the Constitutional provision on Filipinazation of public policy. The WTO opens access to foreign markets, especially its major trading partners,
utility, stated in Section 11, Article XII of the 1987 Philippine Constitution, which limits foreign through the reduction of tariffs on its exports, particularly agricultural and industrial products.
ownership of the capital of a public utility to not more than 40%. Thus, provides new opportunities for the service sector cost and uncertainty associated with
The Court ruled that the term "capital" in Section 11, Article XII of the 1987 Constitution refers exporting and more investment in the country. Tanada also contented that an agreement with
only to shares of stock entitled to vote in the election of directors, and thus in the present the WTO violates Sections 11, 12 and 13 of Article II.
case only to common shares, and not to the total outstanding capital stock (common and The court ruled that it is not a violation of Article II sections 11, 12 and 13 because those
non-voting preferred shares). mentioned are considered Declaration of State Principles and Policies and are not self-
Section 11, Article XII of the Constitution, like other provisions of the Constitution expressly executing which means that an act of legislature is needed to solve the issue and the
reserving to Filipinos specific areas of investment, such as the development of natural available remedy is not judicial but political. WTO does not violate the mandate of the
resources and ownership of land, educational institutions and advertising business, is self- Constitution to develop a self-reliant and independent national economy effectively controlled
executing. by Filipinos. For in fact, WTO allows an exchange on the basis of equality and reciprocity,
frowning only on foreign competition that is unfair.
In case of doubt, the Constitution should be considered self-executing rather than
non-self-executing. Unless the contrary is clearly intended, the provisions of the “Filipino First Policy’ while the Constitution indeed mandates a bias in favor of Filipino
Constitution should be considered self-executing, as a contrary rule would give the goods, services, labor and enterprises, at the same time, it recognizes the need for
legislature discretion to determine when, or whether, they shall be effective. (Manila business exchange with the rest of the world on the bases of equality and reciprocity
Prince Hotel v. GSIS) and limits protection of Filipino enterprises only against foreign competition and trade
practices that are unfair-the Constitution did not intend to pursue isolationist policy.
SELF-EXECUTING/DELEGATION OF PRESIDENT- VILLENA V. SECRETARY OF
INTERIOR Case SELF-EXECUTING – UNLESS THE CONTRARY RULE IS CLEARLY INTENDED,– Oposa
v Factoran Case
This case is about the issue raised by the mayor of Makati City with regard to a case filed
against him for constituting bribery to which the Secretary of Interior ordered his suspension. The petitioners alleged the respondent continued approval of the Timber License
Villena, contended that the Secretary of Interior has no power to suspend him since it is Agreements (TLAs) to numerous commercial logging companies to cut and deforest the
exclusive of his powers as the departmend head. remaining forests of the country. Petitioners request the defendant, his agents,
representatives and other persons acting in his behalf to cancel all existing timber license
The court ruled that the Secretary of Interior has the power to suspend the petitioner by virtue agreements in the country; cease and desist from receiving, accepting, processing, renewing
of the “Doctrine of Qualified Political Agency” which states that all executive and or approving new timber license agreements.
administrative organizations are adjuncts of the agents of the Chief Executive, and except in
CONSTI REVIEWER 11

The Court held that while the right to a balanced and healthful ecology is to be found under candidates. Pamatong argued that the comelec indirectly amended the constitutional
the Declaration of Principles and State Policies and not under the Bill of Rights, it does not provisions on the electoral process and limited the power of the sovereign people to choose
follow that it is less important than any of the civil and political rights enumerated in the latter. their leaders. He invoked the constitutional provision ensuring "equal access to opportunities
Such a right belongs to a different category of rights altogether for it concerns nothing less for public office" where he also implicitly claimed that there is a constitutional right to run for
than self-preservation and self-perpetuation — aptly and fittingly stressed by the petitioners or hold public office and, particularly in his case, to seek the presidency.
— the advancement of which may even be said to predate all governments and constitutions.
As a matter of fact, these basic rights need not even be written in the Constitution for they The court ruled that there is no such right. What is recognized is merely a privilege subject
are assumed to exist from the inception of humankind. to limitations imposed by law. Section 26, article ii of the constitution neither bestows such a
right nor elevates the privilege to the level of an enforceable right. The privilege of equal
Constitutional Law; The complaint focuses on one specific fundamental legal right; access to opportunities to public office may be subjected to limitations. As long as the
The right to a balanced and healthful ecology.—The complaint focuses on one specific limitations apply to everybody equally without discrimination, however, the equal access
fundamental legal right—the right to a balanced and healthful ecology which, for the clause is not violated. Equality is not sacrificed as long as the burdens engendered by the
first time in our nation’s constitutional history, is solemnly incorporated in the limitations are meant to be borne by any one who is minded to file a certificate of candidacy.
fundamental law. In the case at bar, there is no showing that any person is exempt from the limitations or the
burdens which they create.
IN CASE OF DOUBT, SELF-EXECUTING BFAR Employees vs. COA Case
On appeal are the Decision dated April 8, 2005 of respondent Commission on Audit (COA) The “equal access” provision is a subsumed part of article ii of the constitution,
in LAO-N-2005-119 upholding the disallowance by the COA Legal and Adjudication Office entitled “declaration of principles and state policies.” The provisions under the article
(COA-LAO), Regional Office No. VII, Cebu City of the P10,000.00 Food Basket Allowance are generally considered not self-executing, and there is no plausible reason for
granted by BFAR to each of its employees in 1999, and COA Resolution dated August 5, according a different treatment to the “equal access” provision. Like the rest of the
2005, denying petitioners motion for reconsideration of said Decision. policies enumerated in article ii, the provision does not contain any judicially
enforceable constitutional right but merely specifies a guideline for legislative or
The social justice provisions of the Constitution are not self-executing principles ready for executive action. The disregard of the provision does not give rise to any cause of
enforcement through the courts; They are merely statements of principles and policies; To action before the courts.
give them effect, legislative enactment is required. We rule on the issue of constitutionality.
Petitioner invokes the provisions of the 1987 Constitution on social justice to warrant the
grant of the Food Basket Allowance. Time and again, we have ruled that the social justice IN CASE OF DOUBT, SELF-EXECUTING UNLESS THE CONTRARY RULE IS CLEARLY
provisions of the Constitution are not self-executing principles ready for enforcement through INTENDED- Manila Prince Hotel vs. GSIS case
the courts. They are merely statements of principles and policies. To give them effect, This case is about MPH contending that they be given the right to match the bid price of
legislative enactment is required. Renong Berhad over the outsanding shares of Manila Hotel Corporation and assailing GSIS
As we held in Kilosbayan, Incorporated v. Morato, 246 SCRA 540 (1995), the principles for selling the shares to a Malaysian company. The argument of MPH relies on ART XII, Sec
and state policies enumerated in Article II and some sections of Article XII are “not 10 par 2. However, GSIS is contending that such provision of the constitution is not self-
self-executing provisions, the disregard of which can give rise to a cause of action in executing, therefore, not in effect.
the courts. They do not embody judicially enforceable constitutional rights but The court ruled that ART XII, Sec 10 is a self-executing provision (which reads In the grant
guidelines for legislation.” of rights, priveleges, and concessions covering the national economy and patrimony, the
IN CASE OF DOUBT – SELF-EXECUTING – UNLESS THE CONTRARY RULE IS State shall give preference to qualified Filipinos). A provision which is complete in itself and
CLEARLY INTENDED PAMATONG VS. COMELEC, GR 161872, APRIL 13, 2004 becomes operative without the aid of supplementary or enabling legislation, or that which
supplies sufficient rule by means of which the right it grants may be enjoyed or protected, is
Rev. Elly velez pamatong had been disqualified as a candidate for presidency when comelec self-executing. Section 10, second paragraph, Art. XII of the 1987 Constitution is a
limited the number of qualified candidates only to those who can afford to wage a nationwide mandatory, positive command which is complete in itself and which needs no further
campaign and/or are nominated by political parties, declaring him and 35 others as nuisance guidelines or implementing laws or rules for its enforcement.
CONSTI REVIEWER 12

A constitutional provision is self-executing if the nature and extent of the right DOCTRINE OF INCORPORATION-LANTION V. DOJ Case
conferred and the liability imposed are fixed by the constitution itself, so that they can
be determined by an examination and construction of its terms, and there is no This case is about when On June 18, 1999 the Department of Justice received from the
language indicating that the subject is referred to the legislature for action. Department of Foreign Affairs a request for the extradition of private respondent Mark
Jimenez to the U.S. The Grand Jury Indictment, the warrant for his arrest, and other
NON-SELF-EXECUTING - Tañada vs. Angara Case supporting documents for said extradition were attached along with the request.
In Tañada vs. Angara, Wigberto E. Tañada, along with various legislators and non- The court ruled the Constitution of the Philippines adopts the generally accepted principles
government organizations, assails the WTO Agreement for violating the mandate of the 1987 of international law as part of the law of the land, and adheres to the policy of peace, equality,
Constitution to develop a self-reliant and independent national economy effectively controlled justice, freedom, cooperation and amity with all nations.
by Filipinos x x x (to) give preference to qualified Filipinos (and to) promote the preferential
use of Filipino labor, domestic materials and locally produced goods by requiring the The suppression of crime is the concern not only of the state where it is committed
Philippines to place nationals and products of member-countries on the same footing as but also of any other state to which the criminal may have escaped, because it saps
Filipinos and local products. the foundation of social life and is an outrage upon humanity at large, and it is in the
interest of civilized communities hat crimes should not go unpunished
The court rule that the principles in Article II of the Constitution are not intended to be self-
executing principles ready for enforcement through the courts. They are used by the judiciary DOCTRINE OF INCORPORATION – MIJARES VS. RANADA
as aids or as guides in the exercise of its power of judicial review, and by the legislature in - A class suit was filed with the US District Court, District of Hawaii by 10 Filipino Martial law
its enactment of laws. They do not embody judicially enforceable constitutional rights but human rights victim against the estate of the former president Marcos. The petitioners were
guidelines for legislation. required to pay P472,000,000 for filling fee, in which the petitioners only paid P410 as docket
The reasons for denying a cause of action to an alleged infringement of broad and filing fee, notwithstanding the fact that they sought to enforce a monetary amount of
constitutional principles are sourced from basic considerations of due process and damages.
the lack of judicial authority to wade “into the uncharted ocean of social and economic - It is necessary for an action to be filed in order to enforce a foreign judgment, even if such
policy making.” judgment has conclusive effect as in the case of in rem actions, if only for the purpose of
allowing the losing party an opportunity to challenge the foreign judgment, and in order for
the court to properly determine its efficacy. Consequently, the party attacking a foreign
DOCTRINE OF INCORPORATION - Ang Ladlad LGBT Party v COMELEC Case judgment has the burden of proof overcoming the presumption of its validity.
Petitioner is a national organization which represents the lesbians, gays, bisexuals, and - There is no obligatory rule derived from treaties of conventions that requires the Philippines
trans-genders. It filed a petition for accreditation as a party-list organization to public to recognize foreign judgments, or allow a procedure for the enforcement thereof. However,
respondent. However, due to moral grounds, the latter denied the said petition. To buttress generally accepted principles of international law, by virtue of the incorporation clause
their denial, COMELEC cited certain biblical and quranic passages in their decision. It also of the Constitution, form part of the law of the land even if they do not derive from
stated that since their ways are immoral and contrary to public policy, they are considered treaty obligations.
nuisance. In fact, their acts are even punishable under the Revised Penal Code in its Article
201. DOCTRINE OF INCORPORATION; ADHERENCE TO POLICY OF PEACE, FREEDOM,
AMITY – Razon vs Tagitis, GR 182498
In this context, the principle of non-discrimination requires that laws of general application
relating to elections be applied equally to all persons, regardless of sexual orientation. Brief Summary: The case is about General Razon, among other officials of PNP, assailing
Although sexual orientation is not specifically enumerated as a status or ratio for the issuance of Writ of Amparo and the confirmation of the enforced disappearance of Engr.
discrimination in Article 26 of the ICCPR, the ICCPR Human Rights Committee has opined Morced Tagitis under the UN Declaration on Protection of all Persons from Enforced
that the reference to "sex" in Article 26 should be construed to include "sexual orientation." Disappearances.
Additionally, a variety of United Nations bodies have declared discrimination on the basis of Ruling: The Court ruled that underlying every enforced disappearance is a violation of the
sexual orientation to be prohibited under various international agreements. constitutional rights to life, liberty and security which the court should act upon even in the
CONSTI REVIEWER 13

absence of a specific penal law. As a matter of human right and fundamental freedom and economy effectively controlled by Filipino entrepreneurs, it does not impose a
as a policy matter made in the UN Declaration, the ban on enforced disappearance cannot policy of Filipino monopoly of the economic environment.
but have its effects on the country. The Court held further that generally accepted principles
of international law, by virtue of the incorporation clause of the Constitution, form part of the
laws of the land even if they do not derive from treaty obligations. CIVILIAN SUPREMACY, IN RELATION WITH ART VII, SEC 18 – IBP vs Zamora Case

Jurisprudence: The most widely accepted statement of sources of international law - This is a case wherein IBP questioned the constitutionality of the action of the then
today is Article 38(1) of the Statute of International Court of Justice, which provides President Estrada, in efforts to reduce the continuing lawlessness and violence in
that the Court shall apply “international custom, as evidence of a genera; practice some parts of manila through exercising the power vested in him by the virtue of
accepted as law”. Art VII, sec 18, enjoined the PNP, AFP as well as the Marines to controvert the
rising anarchy. Their contention is that such acts makes the PNP, a military
SUFFICIENCY STANDARD TEST – Chiongbian vs Orbos, GR 96754 authority, head, is a violation of the Civilian Supremacy as written in Art II sec 3.
- The court ruled that the deployment of the Marines does not constitute a breach of
Brief Summary: These suits challenge the validity Art. XIX, Section 13 of RA 6734 authorizing the civilian supremacy clause. The calling of the Marines constitutes permissible
the President to merge by administrative determinations the regions remaining after the use of military assets for civilian law enforcement. The local police forces are the
establishment of the Autonomous Region. ones in charge of the visibility patrols at all times, the real authority is still lodged on
Ruling: The Court ruled that there is no abdication by Congress of its legislative power in the chief of PNP.
conferring to the President the power to merge administrative region but there should be - Civilian Supremacy Clause; The deployment of the Marines in the metropolis
sufficient standard provided by Congress to guide the President in the exercise of the power for civilian law enforcement does not constitute a breach of the civilian
granted. The legislative standard need not be expressed; it may simply be implied or supremacy clause.
gathered. It may also be embodied in other statutes on the same subject as that of RA 5435
which purpose was to promote “simplicity, economy and efficiency in the government”. THE DEFENSE OF THE STATE (POSSE COMMITATUS) - People vs. Lagman and Zosa
Jurisprudence: A legislative standard need not be expressed. It may simply be gathered - In People vs. Lagman and Zosa, Tranquilino Lagman and Primitivo De Sosa willfully
or implied. and unlawfully refused to register in the military service notwithstanding the fact that
they had been required to do so. Primitivo de Sosa is fatherless and has a mother
SELF-RELIANT AND INDEPENDENT NATIONAL ECONOMY – Espina v Zamora Case and a brother eight years old to support, and Tranquilino Lagman also has a father
to support, has no military learnings, and does not wish to kill or be killed. In this
- R.A. No. 8762 is being assailed as in breach of the constitutional mandate for the instance, the validity of the National Defense Law, under which the accused were
development of a self-reliant and independent national economy effectively controlled by sentenced, is impugned on the ground that it is unconstitutional
Filipinos, for allowing foreign nationals in engaging in the retail trade business in the - The National Defense Law, in so far as it establishes compulsory military service,
country. R.A. No. 8762 also allows natural-born Filipino citizens who had lost their does not go against this constitutional provision but is, on the contrary, in faithful
citizenship and now reside in the Philippines, to engage in the retail trade business with compliance therewith. The duty of the Government to defend the State cannot be
the same rights as Filipino citizens performed except through an army.
- The duty of the Government to defend the State cannot be performed except
- R.A. No. 8762 is valid. The Court noted that while the Constitution mandates a bias in through an army. To leave the organization of an army to the will of the
favor Filipino goods, services, labor, and local enterprises, it also recognizes the need for citizens would be to make this duty of the Government excusable should
business exchange with the rest of the world on the basis of equality and reciprocity and there be no sufficient men who volunteer to enlist therein.
limits protection of Filipino enterprises only against foreign competition and trade
practices that are unfair.
- National Economy and Patrimony; While Section 19, Article II of the 1987
Constitution requires the development of a self-reliant and independent national
CONSTI REVIEWER 14

POLITICAL QUESTIONS constitutionality of rh law on the ground that it violates the right to religious freedom. The
Vinuya vs. Exec. Secretary, GR 162230, April 28, 2010 petitioners contend that the rh law violates the constitutional guarantee respecting religion as
it authorizes the use of public funds for the procurement of contraceptives. For the petitioners,
Petitioners pray that the Court reconsider its April 28, 2010 decision, and declare: (1) that the the use of public funds for purposes that are believed to be contrary to their beliefs is included
rapes, sexual slavery, torture and other forms of sexual violence committed against the in the constitutional mandate ensuring religious freedom.
Filipina comfort women are crimes against humanity and war crimes under customary
international law; (2) that the Philippines is not bound by the Treaty of Peace with Japan, In the case at bench, it is not within the province of the court to determine whether the use of
insofar as the waiver of the claims of the Filipina comfort women against Japan is concerned; contraceptives or one's participation in the support of modem reproductive health measures
(3) that the Secretary of Foreign Affairs and the Executive Secretary committed grave abuse is moral from a religious standpoint or whether the same is right or wrong according to one's
of discretion in refusing to espouse the claims of Filipina comfort women; and (4) that dogma or belief. For the court has declared that matters dealing with "faith, practice, doctrine,
petitioners are entitled to the issuance of a writ of preliminary injunction against the form of worship, ecclesiastical law, custom and rule of a church ... Are unquestionably
respondents. ecclesiastical matters which are outside the province of the civil courts." the jurisdiction of the
court extends only to public and secular morality. Whatever pronouncement the court makes
The Court denied the Motion for Reconsideration and Supplemental Motion for in the case at bench should be understood only in this realm where it has authority. Stated
Reconsideration for being devoid of merit. The Court cannot grant petitioners’ prayer for the otherwise, while the court stands without authority to rule on ecclesiastical matters, as
writ of preliminary mandatory injunction, citing Marquez v. The Presiding Judge (Hon. Ismael vanguard of the constitution, it does have authority to determine whether the rh law
B. Sanchez), “It is basic that the issuance of a writ of preliminary injunction is addressed to contravenes the guarantee of religious freedom.
the sound discretion of the trial court, conditioned on the existence of a clear and positive
right of the applicant which should be protected. It is an extraordinary, peremptory remedy Verily, the principle of separation of church and state is based on mutual respect.
available only on the grounds expressly provided by law, specifically Section 3, Rule 58 of Generally, the state cannot meddle in the internal affairs of the church, much less
the Rules of Court. Moreover, extreme caution must be observed in the exercise of such question its faith and dogmas or dictate upon it. It cannot favor one religion and
discretion. It should be granted only when the court is fully satisfied that the law permits it discriminate against another. On the other hand, the church cannot impose its beliefs
and the emergency demands it. The very foundation of the jurisdiction to issue a writ of and convictions on the state and the rest of the citizenry. It cannot demand that the
injunction rests in the existence of a cause of action and in the probability of irreparable injury, nation follow its beliefs, even if it sincerely believes that they are good for the country.
inadequacy of pecuniary compensation, and the prevention of multiplicity of suits. Where
facts are not shown to bring the case within these conditions, the relief of injunction should Consistent with the principle that not any one religion should ever be preferred over
be refused.” another, the constitution in the above-cited provision utilizes the term "church" in its
generic sense, which refers to a temple, a mosque, an iglesia, or any other house of
Here, the Constitution has entrusted to the Executive Department the conduct of god which metaphorically symbolizes a religious organization. Thus, the "church"
foreign relations for the Philippines. Whether or not to espouse petitioners' claim against means the religious congregations collectively.
the Government of Japan is left to the exclusive determination and judgment of the Executive
Department. The Court cannot interfere with or question the wisdom of the conduct of
foreign relations by the Executive Department. Accordingly, the Court cannot direct EQUAL ACCESS TO PUBLIC SERVICE; POLITICAL DYNASTIES Pamatong vs.
the Executive Department, either by writ of certiorari or injunction, to conduct foreign COMELEC Case
relations in a certain manner.
- Petitioner Rev. Elly Velez Pamatong filed his Certificate of Candidacy for President on
December 17, 2003. Respondent Commission on Elections (COMELEC) refused to give due
SEPARATION OF CHURCH AND STATE course to petitioner’s Certificate of Candidacy declaring petitioner and thirty-five (35) others
IMBONG VS. OCHOA, GR 204819, APRIL 8, 2014 nuisance candidates who could not wage a nationwide campaign and/or are not nominated
by a political party or are not supported by a registered political party with a national
On december 21, 2012, ra 10354 otherwise known as the rh law was enacted by the constituency. In this Petition For Writ of Certiorari, petitioner seeks to reverse the resolutions
congress. The president placed his imprimatur on said law. Thereafter, challengers from which were allegedly rendered in violation of his right to "equal access to opportunities for
various sectors of society came knocking on the doors of the court, beckoning it to wield the public service" under Section 26, Article II of the 1987
sword that strikes down constitutional disobedience. The petitioners assailed the
CONSTI REVIEWER 15

- There is no constitutional right to run for or hold public office and, particularly, to seek the naturally arouse the interest of an ordinary citizen. // National board examinations
presidency what is recognized is merely a privilege subject to limitations imposed by law. such as the Certified Public Accountant (CPA) Board Exams are matters of public
Implicit in the petitioner’s invocation of the constitutional provision ensuring “equal access to concern; There may be valid reasons to limit access to the Examination Papers in
opportunities for public office” is the claim that there is a constitutional right to run for or hold order to properly administer the exam—more than the mere convenience of the
public office and, particularly in his case, to seek the presidency. There is none. What is examiner, it may well be that there exist inherent difficulties in the preparation,
recognized is merely a privilege subject to limitations imposed by law. Section 26, Article II generation, encoding, administration, and checking of these multiple choice exams
of the Constitution neither bestows such a right nor elevates the privilege to the level of an that require that the questions and answers remain confidential for a limited duration
enforceable right. There is nothing in the plain language of the provision which suggests such
a thrust or justifies an interpretation of the sort. POLITICAL QUESTION DOCTRINE:

- The provisions under the Article are generally considered not self-executing, and The limitation on the power of judicial review to actual cases and controversies carries
there is no plausible reason for according a different treatment to the “equal access” the assurance that the courts will not intrude into areas committed to the other
provision like the rest of the policies enumerated in Article II, the provision does not branches of government.
contain any judicially enforceable constitutional right but merely specifies a guideline Political Question defined: Those questions which, under the Constitution, are to be decided
for legislative or executive action.—The “equal access” provision is a subsumed part by the people in their sovereign capacity, or in regard to which full discretionary authority has
of Article II of the Constitution, entitled “Declaration of Principles and State Policies.” been delegated to the Legislature or executive branch of the Government. It is concerned
The provisions under the Article are generally considered not self-executing, and there with issues dependent upon the wisdom, not legality, of a particular measure.
is no plausible reason for according a different treatment to the “equal access”
provision. Like the rest of the policies enumerated in Article II, the provision does not LOCAL AUTONOMY POLICY:
contain any judicially enforceable constitutional right but merely specifies a guideline
for legislative or executive action. The disregard of the provision does not give rise to Provide the needed impetus and encouragement to the development of local political
any cause of action before the courts. subdivisions (barangays) as "self - reliant communities."

Full Public Disclosure – Antolin vs. Domondon; G.R. No. 165036 July 5, 2010 HUMAN RIGHTS – SIMON V. CHR

Antolin took the 1997 CPA Licensure exam and failed. She questioned the results and - The petitioners in this case issued a “Demolition Notice” of their sari sari store, carenderia,
wanted access to copies of her answer sheets, the questionnaire, the answer keys and an and stalls along the North EDSA. The respondent John Does filed a complaint with the CHR,
explanation of the grading system. She prayed for access under the assumption of Full Public CHR Case No. 90-1580, to stop the demolition. Upon ocular inspection, the stalls were
Disclosure to access the requested documents. demolished prompting the CHR to grant 200,000.00 Pesos as financial assistance and a
warning to desist on the demolishment in favor of the private respondents. Thus, a petition
The Right to Information is not absolute. It is limited only to “matters of public concern.” The to prohibit the CHR from proceeding with the hearing of CHR Case No. 90-1580.
State’s policy of full disclosure is limited to “transactions involving public interest,” and is
“subject to reasonable conditions prescribed by law” There is no clear definition of “public - The court recognized the role of the CHR to protect human rights. However, in this case it
concern”, however, National board examinations such as the CPA Board Exams can be committed an error in imposition of penalty and writ of injunction as the land in question,
considered as matters of public concern. Case REMANDED BACK TO RTC for further adjoins the North EDSA of Quezon City which the Court can take judicial notice of, is a busy
proceedings wherein the PRC may explain the confidentiality of such examinations. national highway. It is noted that a right which is claimed to have been violated is one that
cannot in the first place even be invoked. Therefore, the petition is granted.
Professional Regulation Commission; Right to Information; In determining whether a
particular information is of public concern there is no rigid test which can be applied— - CIVIL RIGHTS: Rights that belong to every citizen of the state or country, or, in a
“public concern” like “public interest” is a term that eludes exact definition, as both wider sense, to all its inhabitants, and are not connected with the organization or
terms embrace a broad spectrum of subjects which the public may want to know, administration of government. They include the rights of property, marriage, equal
either because these directly affect their lives, or simply because such matters protection of the laws, freedom of contract, etc. Or, as otherwise defined civil rights
are rights appertaining to a person by virtue of his citizenship in a state or community.
CONSTI REVIEWER 16

Such term may also refer, in its general sense, to rights capable of being enforced or Philippine Army on the Funeral Honors and Service for President Marcos. Dissatisfied with
redressed in a civil action. the foregoing issuance, the petitioners filed a Petition for Certiorari and Prohibition and
Petition for Mandamus and Prohibition with the Court.
POLITICAL RIGHTS: Rights to participate, directly or indirectly, in the establishment
or administration of government, the right of suffrage, the right to hold public office, The Court upheld what is legal and just, and that is not to deny Marcos of his rightful place
the right of petition and, in general, the rights appurtenant to citizenship vis-a-vis the at the LNMB. The Constitution even though it is a product of our collective history as a people,
management of government. its entirety should not be interpreted as providing guiding principles to just about anything
remotely related to the Martial Law period such as the proposed Marcos burial at the LNMB.
FAMILY AS A BASIC AUTONOMOUS SOCIAL INSTITUTION – Imbong vs. Ochoa
Sec. 1, Art. XI of the Constitution is not a self-executing provision considering that a
Petitioners assail the Constitutionality of the proposed RH Law on several grounds. law should be passed by the Congress to clearly define and effectuate the principle
Petitioners contend that RA 10354 violate Art. II, Secs. 12 and 15. RA 10354 recognizes the embodied therein. At bar, several statutes were invoked but the burial of Marcos at the
right to life of the unborn at the time of “conception” or at the fertilization of the egg. RA 10354 LNMB does not contravene R.A. No. 289, R.A. No. 10368, and the international human rights
against abortion and only pushes for the use of non-abortive contraceptives. RA 10345 also laws cited by petitioners.
provides that the consent of the spouse who seeks to undertake any RH procedures is
sufficient to undergo such procedures. Similarly, any minor who is already a parent or has Women – PT & T vs. NLRC
undergone miscarriage may avail of the aforementioned RH procedures without the consent
of their parents. Grace de Guzman was terminated from her employment at the Philippine Telegraph and
Telephone Company for the reason that she concealed her marital status, an act prohibited
RA 10354 proposes the use of non-abortive contraceptives, or those that DO NOT destroy by the company. Upon learning, the company sent her a memo and terminated de Guzman.
the fertilized egg. However, the use of the term “primarily” in defining abortaficients
(contraceptives which harm the fertilized egg in preventing pregnancy) violates Sec. 12, Art. The court held that petitioner’s policy of not accepting or considering as disqualified from
II. As it paves the way for the approval of the use of such contraceptives. work any woman worker who contracts marriage runs afoul of the test of, and the right
against, discrimination, afforded all women workers by our labor laws and by no less than the
RA 10354 violates Sec. 12, Art. II, with respect to the proviso allowing minors to undergo RH Constitution. Petitioner’s policy is also a violation of the provisions of Article 136 of the Labor
procedures. Sec 12, Art. II places great importance on the role of parents in the upbringing Code on the right of a woman to be free from any kind of stipulation against marriage in
of their children. The right of the parents is superior in the upbringing of their youth as connection with her employment.
compared to the right of the State. It also intrudes on the marital privacy and autonomy as it
contravenes the safeguards of the family as a basic social institution. The mutual decision- Sec. 14, Article II of the 1987 Constitution, expressly recognizes the role of women in
making of the family and the right of the spouses to find a family according to their beliefs is nation-building and commands the State to ensure, at all times, the fundamental
violated by RA 10354 allowing the spouse to undergo its procedures without the consent of equality before the law of women and men.
their partner. CALLADO VS. INTERNATIONAL RICE RESERCH INSTITUTE (IRRI)
Reproductive Health Law; Life Begins at Fertilization This case is about Ernesto Callado, petitionee who was employed as a driver at the IRRI.
One day while driving an IRRI vehicle on an official trip to the NAIA and back to the IRRI,
petitioner figured in an accident. Thereafter, he was terminated that led him to file a complaint
MANDATORY (RATHER THAN DIRECTORY) PROSPECTIVE (RATHER THAN before the Labor Arbiter against IRRI for illegal dismissal. In defense, IRRI invoked its
RETROACTIVE) – OCAMPO VS. ENRIQUEZ immunity from suit by virtue of Art. 3 of P.D. 1620. However, Callado contended that said
immunity may not be invoked because IRRI already waived its immunity by virtue of its
During the campaign period for the 2016 Presidential Election, then candidate Rodrigo R. Memorandum on “Guidelines on the handling of dismissed employee in relation to P.D.
Duterte publicly announced that he would allow the burial former President Ferdinand E. 1620”.
Marcos at the Libingan ng Mga Bayani ("LNMB"). He won the May 9, 2016 elections. Defense
Secretary Delfin N. Lorenzana issued a Memorandum to AFP Chief of Staff regarding the It is undisputed that IRRI is immune from suit and an express waiver by the IRRI’s Director-
interment of former President Marcos. A directive was issued by AFP Rear Admiral to the General is the only way by which the IRRI may abandon its immunity. However, the court
CONSTI REVIEWER 17

ruled that a memorandum which was meant for internal circulation and not as a pledge of constitutional mechanism such as local legislation) or incorporation (mere constitutional
waiver in all cases arising from dismissal of employees cannot be considered the express declaration i.e. treaties).
waiver by the Director-General.
On the other hand, The court ruled that DOH failed to establish that the provisions pertinent
And where the plea of diplomatic immunity is recognized and affirmed by the executive WHA resolutions are customary international law that may be deemed part of the law of the
branch of the government, it is then the duty of the courts to accept the claim of land because for an international law to be considered as customary law, it must be
immunity. established that such rule is being followed by states because they consider it as obligatory
to comply with such rules (opinion juris). The WHO resolutions, although signed by most of
People vs. Dacuycuy the member states, were enforced or practiced by at least a majority of member states unlike
This case is about the public school officials who were charged for violation of RA 4670 or the ICBMS whereby legislature enacted most of the provisions into the law via the Milk Code,
Magna Carta for Public School Teachers. These officials then moved to squash the charges the WHA Resolutions have not been adopted as domestic law nor are they followed in our
against them for lack of jurisdiction and assailed the constitutionality of Sec. 32 of RA 4670 country as well.
for two reasons: (1) the term of imprisonment in the said provision was unfixed and may run WHA Resolutions may be classified as SOFT LAW – non-binding norms, principles
to reclusion perpetua and (2) that it constitutes an undue delegation of legislative power, for and practices that influence state behavior. Soft law is not part of int’l law.
the duration of penalty of imprisonment is left to the discretion of the court.
Section 15. The State shall protect and promote the right to health of the people and
Yes, Republic Act No. 4760 is unconstitutional. Section 32 of Republic Act No. 4670 provides instill health consciousness among them. - Imbong vs. Ochoa
for an indeterminable period of imprisonment, with neither a minimum nor a maximum
duration having been set by the legislative authority. The courts are thus given a wide latitude Republic Act (R.A.) No. 10354, otherwise known as the Responsible Parenthood and
of discretion to fix the term of imprisonment, without even the benefit of any sufficient Reproductive Health Act of 2012 (RH Law), was enacted by Congress on December 21,
standard, such that the duration thereof may range, in the words of respondent judge, from 2012. The petitioners claim that the RH Law violates the right to health because it requires
one minute to the life span of the accused. The court ruled that Section 32 violates the the inclusion of hormonal contraceptives, intrauterine devices, injectables and family
constitutional prohibition against undue delegation of legislative power by vesting in the court products and supplies in the National Drug Formulary and the inclusion of the same in the
the responsibility of imposing duration on the punishment of imprisonment, as if the courts regular purchase of essential medicines and supplies of all national hospitals.
were the legislative department of the government.
The legislative intent in the enactment of the RH Law in this regard is to leave intact the
provisions of R.A. No. 4729. There is no intention at all to do away with it. It is still a good law
and its requirements are still in to be complied with. Thus, the Court agrees with the
Pharmaceutical and Healthcare Association of Philippines vs. Duque observation of respondent Lagman that the effectivity of the RH Law will not lead to the
Petition for certiorari seeking to nullify the Revised Implementing Rules and Regulations unmitigated proliferation of contraceptives since the sale, distribution and dispensation of
(RIRR) of E.O. 51 (Milk Code). Petitioner claims that the RIRR is not valid as it contains contraceptive drugs and devices will still require the prescription of a licensed physician. With
provisions that are not constitutional and go beyond what it is supposed to implement. Milk R.A. No. 4729 in place, there exist adequate safeguards to ensure the public that only
Code was issued by President Cory Aquino under the Freedom Constitution on Oct.1986. contraceptives that are safe are made available to the public. At any rate, it bears pointing
One of the preambular clauses of the Milk Code states that the law seeks to give effect to Art out that not a single contraceptive has yet been submitted to the FDA pursuant to the RH
11 of the Int’l Code of Marketing and Breastmilk Substitutes(ICBMS), a code adopted by the Law. It behooves the Court to await its determination which drugs or devices are declared by
World Health Assembly(WHA). From 1982-2006, The WHA also adopted severe resolutions the FDA as safe, it being the agency tasked to ensure that food and medicines available to
to the effect that breastfeeding should be supported, hence, it should be ensured that nutrition the public are safe for public consumption. Consequently, the Court finds that, at this point,
and health claims are not permitted for breastmilk substitutes. In 2006, the DOH issued the the attack on the RH Law on this ground is premature. Indeed, the various kinds of
assailed RIRR. contraceptives must first be measured up to the constitutional yardstick as expounded herein,
to be determined as the case presents itself.
The court ruled that ICMBS having transformed into domestic law through a local legislation
such as the Milk Code, is considered part of the law of the land because under 1987
Constitution, international law can become domestic law by transformation (through
CONSTI REVIEWER 18

“…. that all inhabitants of Philippine Islands continuing to reside therein, who were LABOR
Spanish subjects on the 11th day of April, 1891, and then resided in said Islands, and
their children born subsequent thereto, shall be deemed end held to be citizens of the JMM Promotion and Management vs. CA (in relation to sec. 3, Article XIII)
Philippines” - Tecson vs. COMELEC  This case is originated from the death of Maricris Sioson in 1991 by which then
This case is about FPJ whose parents were Allan F. Poe and Bessie Kelley. FPJ was born President Corazon Aquino banned the deployment of performing artists to Japan and
to them on 20 August 1939. His parents were married to each other on 16 September 1940. other destinations. This was relaxed however with the introduction of the Entertainment
The father of Allan Poe was Lorenzo Poe and at the time of his death 11 September 1954, Industry Advisory Council which later proposed a plan to POEA to screen and train
Lorenzo Poe was 84 years old. The petitioner Fornier initiated the petition to disqualify FPJ performing artists seeking to go abroad. In pursuant to the proposal POEA and the
and cancel his COC because FPJ was not a natural-born Filipino citizen. Fornier alleged that secretary of DOLE sought a four-step plan to realize the plan which included an Artist’s
his parents were foreigners; his mother was an American and his father, Allan Poe was a Record Book which a performing artist must acquire prior to being deployed abroad.
Spanish national, being the son of Lorenzo Pou, a Spanish subject. Fornier further averred The Federation of Talent Managers of the Philippines assailed the validity of the said
that Allan Poe was a Filipino citizen, he could not have transmitted his Filipino citizenship to regulation as it violated the right to travel, abridge existing contracts and rights and
FPJ, the latter being an illegitimate child of an alien mother. deprives artists of their individual rights.

The court ruled that any conclusion on the Filipino citizenship of Lorenzo Pou could only be  In the case at bar, the challenged Department Order clearly applies to all performing
drawn from the presumption that having died in 1954 at 84 years old, Lorenzo would have artists and entertainers destined for jobs abroad. These orders, we stressed hereinfore,
been born sometime in the year 1970, when the Philippines was under the Spanish rule, and further the Constitutional mandate requiring government to protect our workforce,
that San Carlos, Pangasinan, his place of residence upon his death in 1954, in the absence particularly those who may be prone to abuse and exploitation as they are beyond the
of any other evidence could have well been his place of residence before death, such that physical reach of government regulatory agencies. The tragic incidents must somehow
Lorenzo Pou would have benefited from the en masse Filipinization that the Philippine Bill stop, but short of absolutely curtailing the right of these performers and entertainers to
had effected in 1902. That citizenship of Lorenzo Pou if acquired, would thereby extend to work abroad, the assailed measures enable our government to assume a measure of
his son, Allan F. Poe father of FPJ. control.

Under 1935 Constitution, the fundamental law prevailed on the day, month and year of  Apart from the State’s police power, the Constitution itself mandates government to
birth of FPJ, it stated there that those whose fathers are citizen of the Philippines extend the fullest protection to our overseas workers.
regardless of whether such children are legitimate or illegitimate are also citizens of
the Philippines.
RIGHT TO HEALTH – MMDA VS. RESIDENTS OF MANILA BAY
RIGHT TO BALANCE AND HEALTHFUL ECOLOGY – Oposa vs Factoran, GR 101083
- The respondents Concerned Citizens of Manila Bay filed a complaint before the RTC
Brief Summary: This case is about a petition for the cancellation of Timber License against several government agencies including the petitioner for the cleanup, rehabilitation,
Agreements (TLAs) asserting that the granting of TLAs is a violation of the right to a balanced and protection of the Manila Bay. In said complaint, the water quality of the Manila Bay had
and healthful ecology. falled way below the allowable standards set by the law, also stating that the government
Ruling: The Court held that the right to a balanced and healthful ecology belongs to a different agencies must be held liable and be collectively ordered to clean up Manila Bay because it
category of rights altogether for it concerns nothing less that self-preservation and self- presents a clear and present danger to public health.
perpetuation the advancement of which may even be said to predate all governments and - The Court ruled in favor of the respondents, stating that Sec. 17 does not in any way state
constitutions. It carries with it the correlative duty to refrain from impairing the environment. that the government agencies concerned ought to confine themselves to the containment,
Thus, the right to a balanced and healthful ecology is as clear as the DENR’s duty to protect removal and cleaning operations with a specific pollution incident occurs. It requires them to
and advance the sad right under its mandate and by virtue of its powers and functions under act even in the absence of a specific incident.
EO 192 and administrative Code of 1987.
Jurisprudence: The complaint focuses on one specific fundamental legal right – the
right to a balanced and healthful ecology.
CONSTI REVIEWER 19

- The Court stated that the right to a balanced and healthful ecology need not even be written judgments in utilizing public funds for local development. With PDAF, a Congressman can
in the Constitution for it is assumed, like other civil and political rights guaranteed in the Bill simply bypass the local development council and initiate projects on his own, and even take
of Rights, to exist from the inception of mankind and it is an issue of transcendental sole credit for its execution. Indeed, this type of personality-driven project identification has
not only contributed little to the overall development of the district, but has even contributed
to "further weakening infrastructure planning and coordination efforts of the government."
LOCAL AUTONOMY – Basco vs Pagcor Case The Court observes that this concept of legislator control underlying the CDF and PDAF
This case is about the group of Basco assailing the constitutionality of the charter of Philippine conflicts with the functions of the various Local Development Councils (LDCs). Considering
Amusment and Gaming Corporation (PAGCOR). Their contention is that it constitutes a that LDCs are instrumentalities whose functions are essentially geared towards managing
waiver of the right of the City of Manila to improve taxes and legal fees; and that the local affairs, their programs, policies and resolutions should not be overridden nor duplicated
exemption clause in PD 1869 is violative of constitutional principle of Local Autonomy. by individual legislators, who are national officers that have no law-making authority except
only when acting as a body. Thus, insofar as individual legislators are authorized to intervene
The court ruled that the City of Manila, being a mere Municipal Corporation, has no inherent in purely local matters and thereby subvert genuine local autonomy, the 2013 PDAF Article
right to impose taxes. Its charter was created by Congress, therefore subject to its control. as well as all other similar forms of Congressional Pork Barrel is deemed unconstitutional.
Also, local governments have no power to tax instrumentalities of the National Government
and PAGCOR is one of them. The Court finds an inherent defect in the system which actually belies the avowed
intention of "making equal the unequal." In particular, the Court observes that the
Local governments have no power to tax instrumentalities of the National gauge of PDAF and CDF allocation/division is based solely on the fact of office,
Government; PAGCOR, being an instrumentality of the Government, is therefore without taking into account the specific interests and peculiarities of the district the
exempt from local taxes. legislator represents. In this regard, the allocation/division limits are clearly not based
on genuine parameters of equality, wherein economic or geographic indicators have
LOCAL AUTONOMY - Lina vs. Pano Case been taken into consideration. As a result, a district representative of a highly-
In Lina vs. Pano, Tony Calvento was denied of a mayor’s permit to operate a lotto outlet by urbanized metropolis gets the same amount of funding as a district representative of
Mayor Calixto Cataquiz of San Pedro, Laguna pursuant to a resolution by the Sangguniang a far-flung rural province which would be relatively "underdeveloped" compared to the
Panlalawigan of Laguna that declared, in sum, local legislator’s vehement opposition of any former. To add, what rouses graver scrutiny is that even Senators and Party-List
forms of gambling, including lotto. This is despite the fact that lotto is a game of chance duly Representatives – and in some years, even the Vice-President – who do not represent
authorized by the national government through an Act of Congress. any locality, receive funding from the Congressional Pork Barrel as well. These
certainly are anathema to the Congressional Pork Barrel‘s original intent which is "to
The court ruled that, as a policy statement expressing the local government's objection to the make equal the unequal." Ultimately, the PDAF and CDF had become personal funds
lotto, such resolution is valid. This is part of the local government's autonomy to air its views under the effective control of each legislator and given unto them on the sole account
which may be contrary to that of the national Government's. However, this freedom to of their office.
exercise contrary views does not mean that local governments may actually enact ordinances
that go against laws duly enacted by Congress.
POLITICAL QUESTION/LOCAL AUTONOMY – BELGICA V. OCHOA
The principle of local autonomy under the 1987 Constitution simply means
“decentralization”. It does not make local governments sovereign within the state or -The case involves the question of the constitutionality of Priority Development Assistance
an “imperium in imperio.” Fund (PDAF), a congressional pork barrel on a lump sum basis of budgeting. PDAF, being
budgetary in nature and as a subject of the case it draws the question of jurisdiction whether
LOCAL AUTONOMY the Court should take cognizance of the issue.
Belgica vs. Ochoa, GR 208566, Nov. 19, 2013
- On Political Question Doctrine: The intrinsic constitutionality of the "Pork Barrel System" is
In the cases at bar, petitioners assail the Constitutionality of the Pork Barrel System. They not an issue dependent upon the wisdom of the political branches of government but rather
contend that the Congressional Pork Barrel goes against the constitutional principles on local a legal one which the Constitution itself has commanded the Court to act upon. Scrutinizing
autonomy since it allows district representatives, who are national officers, to substitute their
CONSTI REVIEWER 20

the contours of the system along constitutional lines is a task that the political branches of RA 9189 THE OVERSEAS ABSENTEE VOTING ACT OF 2003 – Nicolas-Lewis vs
government are incapable of rendering precisely because it is an exercise of judicial power. COMELEC, GR 162759
On Local Autonomy Policy: The Congressional Pork Barrel goes against the constitutional - Brief Summary: The case is about “duals” or dual citizens who retained or reacquired
principles on local autonomy since it allows district representatives, who are national officers, Philippine citizenship under RA 9225, the Citizen Retention and Reacquisition Act of 2003
to substitute their judgments in utilizing public funds for local development. praying to be allowed to vote under mechanisms provided under the Overseas Absentee
Voting Act of 2003.
- Ruling: The Court held that considering the unison intent of the Constitution and RA 9189
(EN MASSE FILIPINIZATION) – CO VS. HRET and the expansion of scope of that law with the passage of RA 9225, the irresistible
conclusion is that “duals” may now exercise the right of suffrage thru the absentee voting
Jose Ong Jr. was born and raised in Nothern Samar by his naturalized Filipino father and scheme and as overseas absentee voters.
natural – born Filipino mother. Laoang, Northern Samar where Ong grew up and chose to - Jurisprudence: There is no provision in the dual citizenship law – RA 9225 –
live even though he went away to study. When he run for congressional election for the requiring “duals” to actually establish residence and physically stay in the
second district of Northern Samar, he was proclaimed the winner and petitioners argued that Philippines first before they can exercise the right to vote – on the contrary, RA
he was not a natural – born Filipino and not a resident of Laoang, Northern Samar. 9225, in implicit acknowledgment that “duals” are most likely non-resident, grants
under its Section 5(1) the same right of suffrage as that granted an absentee voter
It was ruled that when protestee was only nine years of age, his father, Jose Ong Chuan under RA 9189. It cannot be overemphasized that RA 9189 aims, in essence, to
became a naturalized Filipino. Section 15 of the Revised Naturalization Act squarely applies enfranchise as much as possible all overseas Filipinos, who, save for residency
its benefit to him for he was then a minor residing in this country. Concededly, it was the law requirements exacted of an ordinary voter under ordinary conditions, are qualified
itself that had already elected Philippine citizenship for protestee by declaring him as such to vote.
and it would be absurd to ask Jose Ong to elect the citizenship he had already acquired since
his childhood. RA 9225- POE-LLAMANZARES VS. COMELEC CASE
Any election of Philippine citizenship on the part of private respondent Jose Ong, Jr. Mary Grace Natividad S. Poe-Llamanzares was found abandoned as a newborn infant.
would not only have been superfluous but would also have resulted in absurdity Ronald Allan Kelley Poe (a.k.a. Fenando Poe, Jr.) and Jesusa Sonora Poe (a.k.a. Susan
considering that it was the law itself that had already elected Philippine citizenship for Roces) filed a petition for her adoption with the Municipal Trial Court which granted their
him petition Having reached the age of eighteen (18) years in 1986, petitioner registered as a
voter with the local COMELEC Office in San Juan City. Initially, the petitioner enrolled and
R.A. NO. 9189 – OVERSEAS ABSENTEE VOTING LAW – VELASCO VS. COMELEC pursued a degree in Development Studies at the University of the Philippines but she opted
- Petioner Velasco is a natural-born Filipino who applied for dual citizenship under RA 9225. to continue her studies abroad and left for the United States of America (U.S.) Petitioner
He then applied for registration as voter in Pampanga, which he filed a petition to be included married Teodoro Misael Daniel V. Llamanzares, a citizen of both the Philippines and the
in the list of voters in the municipality as well as his COC for the mayor position. U.S. Desirous of being with her husband who was then based in the U.S., the couple flew
back to the U.S. Petitioner became a naturalized American citizen. Petitioner came home to
- The right of dual citizens who vote as absentee voters pertain only to the election of national the Philippines on 24 May 2005 and without delay, secured a Tax Identification Number from
officials, specifically: the president, vice-president, the senators, and the party-list the Bureau of Internal Revenue. Her three (3) children immediately followed while her
representatives. Thus, the local elections are not included. husband was forced to stay in the U.S. to complete pending projects as well as to arrange
the sale of their family home there but subsequently followed them in the Philippines. On 7
- Sec. 88 of RA 6713 provides that it is the obligation of an employee to submit a sworn July 2006, petitioner took her Oath of Allegiance to the Republic of the Philippines pursuant
statement, as the public has the right to know the employee’s assets, liabilities, and to Republic Act (R.A.) No. 9225 or the Citizenship Retention and Re-acquisition Act of 2003.
net worth and financial and business interests. Sec. 11 of the same law prescribes the Again, petitioner registered as a voter of Barangay Santa Lucia, San Juan City. On 6 October
criminal and administrative penalty for the violation of any provision thereof. 2010, Petitioner executed an "Affidavit of Renunciation of Allegiance to the United States of
America and Renunciation of American Citizenship" before a notary public in satisfaction of
the legal requisites stated in Section 5 of R.A. No. 9225. On 2 October 2012, the petitioner
CONSTI REVIEWER 21

filed with the COMELEC her Certificate of Candidacy (COC) for Senator for the 2013 FOUNDLINGS AS CITIZENS UNDER INTERNATIONAL LAW
Elections wherein she obtained the highest number of votes and was proclaimed Senator on
16 May 2013. On 15 October 2015, petitioner filed her COC for the Presidency for the May Poe-Llamanzares vs. Comelec
2016 Elections  This case is about the original petition filed by Estrella Elamparo to deny due course or
The issue before the COMELEC is whether or not the CoC of petitioner should be denied due cancel COC of Mary Grace Natividad S. Poe-Llamanzares on the issue of citizenship.
course or cancelled “on the exclusive ground” that she made in the certificate a false material Elamparo argued that petitioner cannot be considered as a natural-born Filipino on
representation. The exclusivity of the ground should hedge in the discretion of the COMELEC account of the fact that she was a foundling. Elamparo claimed that international law
and restrain it from going into the issue of the qualifications of the candidate for the position, does not confer natural-born status and Filipino citizenship on foundlings. Following this
if, as in this case, such issue is yet undecided or undetermined by the proper authority. The line of reasoning, petitioner is not qualified to apply for reacquisition of Filipino
COMELEC cannot itself, in the same cancellation case, decide the qualification or lack thereof citizenship under R.A. No. 9225 for she is not a natural-born Filipino citizen to begin
of the candidate. As presently required, to disqualify a candidate there must be a declaration with. Even assuming arguendo that petitioner was a natural-born Filipino, she is
by a final judgment of a competent court that the candidate sought to be disqualified “is guilty deemed to have lost that status when she became a naturalized American
of or found by the Commission to be suffering from any disqualification provided by law or the citizen. According to Elamparo, natural-born citizenship must be continuous from birth.
Constitution.”—Clearly, the amendment done in 2012 is an acceptance of the reality of  The court ruled that in the matter of the citizenship and residence of petitioner for her
absence of an authorized proceeding for determining before election the qualifications of candidacy as President of the Republic, the questioned Resolutions of the COMELEC
candidate. Such that, as presently required, to disqualify a candidate there must be a in Division and En Banc are, one and all, deadly diseased with grave abuse of
declaration by a final judgment of a competent court that the candidate sought to be discretion from root to fruits. The petition is granted.
disqualified “is guilty of or found by the Commission to be suffering from any disqualification
provided by law or the Constitution.”  Foundlings are likewise citizens under international law; The common thread of the
UDHR, UNCRC and ICCPR is to obligate the Philippines to grant nationality from birth
If a candidate cannot be disqualified without a prior finding that he or she is suffering from a and ensure that no child is stateless; It is a generally accepted principle of international
disqualification “provided by law or the Constitution,” neither can the certificate of candidacy law to presume foundlings as having been born of nationals of the country in which the
(CoC) be cancelled or denied due course on grounds of false representations regarding his foundling is found.
or her qualifications, without a prior authoritative finding that he or she is not qualified, such
prior authority being the necessary measure by which the falsity of the representation can be CITIZENSHIP ONCE LOST MAY BE REACQUIRED - Altajeros vs. Comelec Case
found. Insofar as the qualification of a candidate is concerned, Rule 25 and Rule 23 are
flipsides of one to the other. Both do not allow, are not authorizations, are not vestment of In Altajeros vs. Comelec, Ciceron P. Altarejos was disqualified from running as mayor of San
jurisdiction, for the COMELEC to determine the qualification of a candidate. The facts of Jacinto, Masbate due to his naturalization as an American citizen although he avers that he
qualification must beforehand be established in a prior proceeding before an authority properly has already reacquired his Filipino citizenship through repatriation pursuant to R.A. No. 8171.
vested with jurisdiction. The prior determination of qualification may be by statute, by executive The court ruled that, in addition to taking the Oath of Allegiance to the Republic of the
order or by a judgment of a competent court or tribunal. If a candidate cannot be disqualified Philippines, the registration of the Certificate of Repatriation in the proper civil registry and
without a prior finding that he or she is suffering from a disqualification “provided by law or the the Bureau of Immigration is a prerequisite in effecting the repatriation of a citizen under R.A.
Constitution,” neither can the certificate of candidacy be cancelled or denied due course on No. 8171; and that repatriation retroacted to the application.
grounds of false representations regarding his or her qualifications, without a prior
authoritative finding that he or she is not qualified, such prior authority being the necessary In addition to taking the Oath of Allegiance to the Republic of the Philippines, the
measure by which the falsity of the representation can be found. The only exception that can registration of the Certificate of Repatriation in the proper civil registry and the Bureau
be conceded are self-evident facts of unquestioned or unquestionable veracity and judicial of Immigration is a prerequisite in effecting the repatriation of a citizen.
confessions. Such are, anyway, bases equivalent to prior decisions against which the falsity
of representation can be determined. R.A. 9139 – THE ADMINISTRATIVE NATURALIZATION LAW OF 2000 – Frivaldo v
COMELEC Case
Frivaldo was proclaimed governor-elect of the province of Sorsogon, and assumed office in
due time. The League of Municipalities, Sorsogon Chapter, represented by its President,
CONSTI REVIEWER 22

Salvador Estuye, who was also suing in his personal capacity, filed with the Commission on Second Division disqualifying him from running for the position of Vice-Mayor of
Elections a petition for the annulment of Frivaldo on the ground that he was not a Filipino Catarman, Camiguin, in the 14 May 2007 National and Local Elections, on the ground
citizen, having been naturalized in the United States. that he failed to make a personal renouncement of his United States (US) citizenship.
This is in connection with the respondent Rogen T. Dal who initially filed a Petition for
Petitioner claims that he has reacquired Philippine citizenship by virtue of valid repatriation. Disqualification before the COMELEC Provincial Office in Camiguin against petitioner,
He claims that by actively participating in the local elections, he automatically forfeited arguing that the latter failed to renounce his US citizenship, as required under Section
American citizenship under the laws of the United States of America. The Court stated that 5(2) of Republic Act No. 9225.
the alleged forfeiture was between him and the US. If he really wanted to drop his American
citizenship, he could do so in accordance with CA No. 63 as amended by CA No. 473 and  The court ruled that the rules on citizenship qualifications of a candidate must be strictly
PD 725. Philippine citizenship may be reacquired by direct act of Congress, by naturalization, applied. If a person seeks to serve the Republic of the Philippines, he must owe his
or by repatriation. loyalty to this country only, abjuring and renouncing all fealty and fidelity to any other
state. The application of the constitutional and statutory provisions on disqualification is
Constitutional Law; Citizenship; Naturalization; Repatriation; Petitioner’s loss of his not a matter of popularity. Petitioner is disqualified to run for the position of Vice-Mayor
naturalized American citizenship did not and could not have the effect of automatic of Catarman, Camiguin in the 14 May 2007 National and Local Elections, and if
restoration of his Philippine citizenship proclaimed, cannot assume the Office of Vice-Mayor of said municipality by virtue of
RA. 8189 - VOTER'S REGISTRATION ACT OF 1996 such disqualification.

Akbayan vs. Comelec, GR 147066, Mar. 26, 2001  Section 5(2) of Republic Act No. 9225 compels natural-born Filipinos, who have been
naturalized as citizens of a foreign country, but who reacquired or retained their
 This is a case wherein the herein petitioners, - representing the youth sector, invoking Philippine citizenship (1) to take the oath of allegiance under Section 3 of Republic Act
their right, seeking to direct the Commission on Elections (COMELEC) to conduct a No. 9225 and (2) for those seeking elective public offices in the Philippines, to
special registration before the May 14, 2001 General Elections, of new voters ages 18 additionally execute a personal and sworn renunciation of any and all foreign
to 21. According to petitioners, around four million youth failed to register on or before citizenship.
the December 27, 2000 deadline set by the respondent COMELEC under Republic Act
No. 8189 (Voter's Registration Act of 1996). POLICY AGAINST DUAL ALLEGIANCE SO V. REPUBLIC Case

 The court ruled that the COMELEC in denying the request of petitioners to hold a - This case is about the filing of petition for naturalization under Commonwealth Act No.
special registration, acted within the bounds and confines of the applicable law on the 473 (Judicial Act for Naturalization) by Edison So, a Chinese origin. He attached to the
matter - Section 8 of R.A. 8189. In issuing the assailed Resolution, respondent petition a Joint Affidavit of Atty. Adasa, Jr. and Mark Salcedo as his character
COMELEC simply performed its constitutional task to enforce and administer all laws witnesses. RTC granted such petition. However, the OSG appealed to CA arguing that
and regulations relative to the conduct of an election, inter alia, questions relating to the these 2-character witnesses have not qualified as character witnesses and that So is
registration of voters; evidently, respondent COMELEC merely exercised a prerogative not qualified to be admitted as citizen of the Philippines. So, on the other hand, averred
that chiefly pertains to it and one which squarely falls within the proper sphere of its that he graduated cum laude from UST with the degree of Bachelor of Science in
constitutionally mandated powers. Pharmacy and was in his second year as a medical student in UST. He averred that
requirements for naturalization under C.A. 473, as amended, had been relaxed after
The court held that Section 8 of R.A. 8189 applies in the present case, for the purpose of the Philippine government entered into diplomatic relations with China and further
upholding the assailed COMELEC Resolution and denying the instant petitions, considering relaxed by R.A. No. 9139. CA set aside the ruling of the RTC and dismissed So’s
that the aforesaid law explicitly provides that no registration shall be conducted during the petition for naturalization
period starting one hundred twenty (120) days before a regular election
- The court ruled that C.A. 473 shall apply. First, R.A. 9139 and C.A. 473 are separate
POLICY AGAINST DUAL ALLEGIANCE Jacot vs. Dal and distinct laws. R.A. 9139 covers native-born aliens who lived here in the Philippines
all their lives, who never saw any country, and all along thought that they were Filipinos.
 This case is about petitioner Nestor A. Jacot assailing the Resolution of the Commission On the other hand, C.A. 473 covers all aliens regardless of class. A native-born alien
on Elections (COMELEC) En Banc in affirming the Resolution of the COMELEC
CONSTI REVIEWER 23

then has the choice to apply for judicial or administrative naturalization. Since So Citizenship Permanent, Amending for the Purpose Commonwealth Act No. 63, As Amended,
applied by judicial act, then C.A. 473 shall govern. and for Other Purposes." Petitioner avers that Rep. Act No. 9225 is unconstitutional as it
violates Section 5, Article IV of the 1987 Constitution that states, "Dual allegiance of citizens
- R.A. 9139 and C.A. 473 are separate and distinct laws. R.A. 9139 covers native- is inimical to the national interest and shall be dealt with by law." Petitioner contends that
born aliens who lived here in the Philippines all their lives, who never saw any Rep. Act No. 9225 cheapens Philippine citizenship. He avers that Sections 2 and 3 of Rep.
country, and all along thought that they were Filipinos. On the other hand, C.A. Act No. 9225, together, allow dual allegiance and not dual citizenship.
473 covers all aliens regardless of class.
The Office of the Solicitor General (OSG) claims that Section 2 merely declares as a state
importance with intergenerational implications. policy that "Philippine citizens who become citizens of another country shall be deemed not
POLICY AGAINST DUAL ALLEGIANCE – Mercado vs Manzano, GR 135083 to have lost their Philippine citizenship." The OSG further claims that the oath in Section 3
does not allow dual allegiance since the oath taken by the former Filipino citizen is an
- Brief Summary: This case is about Mercado assailing the fitness of Manzano to run in effective renunciation and repudiation of his foreign citizenship. The fact that the applicant
the position of Vice Mayor contending that Manzano have dual citizenship and therefore taking the oath recognizes and accepts the supreme authority of the Philippines is an
have dual allegiance. unmistakable and categorical affirmation of his undivided loyalty to the Republic.
- Ruling: The court ruled that Dual Allegiance and Dual Citizenship are not totally the
same. Also, what is prohibited in the Constitution as per Art IV Sec 5, is only Dual From the excerpts of the legislative record, it is clear that the intent of the legislature
Allegiance. Manzano being of, Dual Citizenship, has no need to secure affidavit of in drafting Rep. Act No. 9225 is to do away with the provision in Commonwealth Act
renunciation of American Citizenship it being already in part with the Oath he took in No. 635 which takes away Philippine citizenship from natural-born Filipinos who
filing for his candidacy. become naturalized citizens of other countries. What Rep. Act No. 9225 does is allow
- Jurisprudence: Dual citizenship is different from dual allegiance, the phrase “dual dual citizenship to natural-born Filipino citizens who have lost Philippine citizenship
citizenship” in Republic Act No. 7160, Sec 40(d) and in Republic Act No. 7854, by reason of their naturalization as citizens of a foreign country. On its face, it does
Sec 20 must be understood as referring to “dual allegiance”. not recognize dual allegiance. By swearing to the supreme authority of the Republic,
the person implicitly renounces his foreign citizenship. Plainly, from Section 3, Rep.
POLICY AGAINST DUAL ALLEGIANCE – LOPEZ VS. COMELEC Act No. 9225 stayed clear out of the problem of dual allegiance and shifted the burden
of confronting the issue of whether or not there is dual allegiance to the concerned
The petitioner Lopez was a candidate for the position of barangay chairman in Iloilo city, foreign country. What happens to the other citizenship was not made a concern of
when the respondent filed a petition praying for the disqualification of the petitioner on the Rep. Act No. 9225.
ground that he is an American citizen, hence ineligible to run for public office. The petitioner
argued that he is a dual citizen and the same time an American by virtue of RA 9225. RENDERING SERVICE TO FOREIGN ARMED FORCES- Bengson III vs HRET Case
The COMELEC ruled that the petitioner was not able to regain his Filipino citizenship as This is a case wherein the qualifications of Bengson III as a member of the House of
provided by law. Although the petitioner was able to re-acquire his Filipino citizenship, the Representatives were assailed on the grounds that he is not a natural-born citizen as required
law provides that should one seek elective public office, he should first make a personal or ART VI sec 6 of the Constitution. They are contending that the Repatriation effects on him is
sworn renunciation of any and all foreign citizenship before any public officer authorized to the same as Naturalization, therefore, he is only a naturalized Filipino citizen.
administer an oath.
The court held that what is lost is what is regained. Repatriation results in the recovery of the
Those seeking elective public office in the Philippines must meet the requirements for original nationality. This means that a naturalized Filipino who lost his citizenship will be
holding such office, and make a personal and sworn renunciation of any and all foreign restored to his prior status as a naturalized Filipino citizen. On the other hand, if he was
citizenship before any public officer authorized to administer an oath. originally a natural-born citizen before he lost his Philippine citizenship, he will be restored to
POLICY AGAINST DUAL ALLEGIANCE (RA 9225) his former status as a natural-born Filipino.
Calilung vs. Secretary of Justice, GR 160869, May 11, 2007
Petitioner filed an action for prohibition to stop respondent from implementing Republic Act
No. 9225, entitled "An Act Making the Citizenship of Philippine Citizens Who Acquire Foreign
CONSTI REVIEWER 24

REPATRIATION RESULTS IN THE RECOVERY OF THE ORIGINAL NATIONALITY. (RESIDENCE QUALIFICATION) – FAYPON VS. QUIRINO
RIGHT TO SUFFRAGE- GMA NETWORK V. COMELEC Respondent was born in Caoayan, Ilocos Sur. He came to Manila to pursue his studies, went
to United States for the same purpose, returned to the Philippines and engaged in the
- The petitioners in this case questions the constitutionality of Sec. 9(A) of the COMELEC newspaper work in Manila, and Iloilo. When he ran for the office of Provincial Governor of
Resolution No. 9615. Such resolution by the COMELEC aims to “level the playing field” by Ilocos Sur, he was proclaimed by the provincial board of canvassers as the governor. A
changing the air time limitations from a per station basis to an aggregate basis. petition for quo warranto was filed by the petitioner on the ground of respondent's ineligibility
- It was ruled by the Court that such section in the COMELEC resolution is unreasonable and for the said office because of alleged lack of residence.
arbitrary thus restricting the ability of the candidates and political parties to reach out and The court ruled that mere absence from one's residence of origin—domicile—to pursue
communicate with the people. It was also noted that it also have affected the right to suffrage studies, engage in business, or practice his avocation, is not sufficient to constitute
by the people on the basis that they won’t be properly communicated with by the candidates abandonment or loss of such residence. The determination of a person's legal residence or
of the election. domicile largely depends upon intention which may be inferred from his' acts, activities and
- Fundamental to the idea of a democratic and republican state is the right of the utterances.
people to determine their own destiny through the choice of leaders they may have in Residence; mere absence does not constitute abandonment thereof. A previous
government. Thus, the primordial importance of suffrage and the concomitant right of registration as voter in a municipality other than that in which he is elected is not
the people to be adequately informed for the intelligent exercise of such birthright. sufficient to constitute abandonment or loss of his residence of origin
POE-LLMANZARES VS. COMELEC RESIDENCE QUALIFICATION – GALLEGO VS. VERRA;
Grace Poe was born in 1968., found as newborn infant in Jaro, Iloilo and was legally adopted Pedro Gallego was elected as Municipal Mayor of Abuyog, Leyte which he was a native of.
by Ronald Allan Kelly Poe (FPJ) And Jesus Sonora Poe (Susan Roces) in 1974. She Verra, his unsuccessful opponent, assailed Gallego’s election citing that Gallego was not a
immigrated to the US in 1991 after her marriage to Theodore Llamanzares who was then resident of Abuyog, on the account that he left to work outside of Abuyog for some years.
based at the US. Grace Poe then became a naturalized American citizen in 2001. On
December 2004, she returned to the Philippines due to his father’s deteriorating medical The Supreme Court held that even if Gallego had lost his residence or domicile in Abuyog,
condition. FPJ then eventually died on February 3, 2005 that made Grace Poe quit her job in he later reacquired it as he clearly announced his intention to return to Abuyog on several
the US to be with her grieving mother and finally went home for good to the Philippines on occasions. There was no abandonment of his domicile in Abuyog as he left his family there
May 24, 2005. On July 18, 2006, the Bureau of Immigration granted her petition declaring and frequently visited them for months at a time.
that she had reacquired her Filipino citizenship under RA 9225. She registered as a voter
and obtained a new Philippine Passport. In 2010, she renounced her American citizenship to Elections; Candidate for Municipal Mayor; Domicile How Acquired.—In order to
satisfy the RA 9225 requirements as to Reacquisition of Filipino Citizenship. From then on, acquire a domicile by choice, there must concur (1) residence or bodily presence in the new
she stopped using her American passport. When she filed her COC, the Comelec en banc locality, (2) (animus manendi) an intention to remain there, and (3) (animus non revertendi)
cancelled her candidacy on the ground that she failed to meet the residence requirements an intention to abandon the old domicile. The purpose to remain in or at the domicile of choice
for the position. must be for an indefinite period of time. The acts of the person must conform with his purpose.
The change of residence must be voluntary; the residence at the place chosen for the
The court ruled that Grace Poe satisfied the 10-year residency because she satisfied the domicile must be actual; and to the fact of residence there must be added the animus
requirements of ANIMUS MANENDI (intent to remain permanently) coupled with ANIMUS manendi
NON REVERTENDI (intent of not returning to US) in acquiring a new domicile in the
Philippines. Starting May 24,2005, upon returning to the Philippines, Grace Poe presented AMENDMENT V. REVISION- DEFENSOR V. COMELEC Case
overwhelming evidence of her actual stay and intent to abandon permanently her domicile in - This case is about the petition of “initiative” headed by Atty. Jesus Delfin to amend the
the US, coupled with her eventual application to reacquire Filipino Citizenship under RA constitution to lift the terms of elective officials by people’s initiatve. Therafter, Miriam
9225. Hence, her candidacy for Presidency was granted by the SC. Santiago with other petitioners filed a special civil action for prohibition contending that
what Delfin wants is not an amendment and is considered as a REVISION.
CONSTI REVIEWER 25

- The court ruled in that RA NO. 6735 intended to include the system of Initiative on PUBLIC OFFICE IS PUBLIC TRUST – Abakada vs Purisima Case
amendments to the constitution but is unfortunately inadequate to cover that system. And
also, THE PROVISION IS NOT SELF-EXECUTORY, therefore and act of - This is a case wherein Abakada, a party list composed of teachers, assails the
congress/legislation is needed to suffice the issue. The people cannot exercise if the constitutionality of. R.A. 9335 also known as the Attrition Act over which optimizes
Congress, for whatever reason does not provide for its implementation. the revenue-generation capability and collection of the Bureau of Internal Revenue
(BIR) and the Bureau of Customs (BOC) through encouraging BIR and BOC
The right of the people to directly propose amendments to the constitution through officials and employees to exceed their revenue targets by providing a system of
the system of Initiative would remain entombed in the cold niche of the rewards and sanctions through the creation of a Rewards and Incentives Fund
Constitution until Congress provides for its implementation (Fund) and a Revenue Performance Evaluation Board (Board). They are
contending that through establishment of a system of rewards and sanctions, it
AMENDMENT V REVISION – Lambino v COMELEC Case transforms the officials and employees of the BIR and the BOC into mercenaries
- Lambino et al filed a petition with the COMELEC to hold a plebiscite thru initiative and bounty hunters” as they will do their best only in consideration of such rewards.
petition to change the 1987 Constitution and to shift the present Bicameral-Presidential - The court ruled that although public service is its own reward, nevertheless, public
system to a Unicameral-Parliamentary form of government. The Lambino Group officers may by law be rewarded for exemplary and exceptional performance. A
alleged that their petition had the support of 6,327,952 individuals constituting at least system of incentives for exceeding the set expectations of a public office was seen
twelve per centum (12%) of all registered voters, with each legislative district as not anathema to the concept of public accountability. In fact, the law recognized
represented by at least three per centum (3%) of its registered voters. and reinforced the dedication to duty, industry, efficiency, and loyalty to the public
service of deserving government personnel.
- The Court held that the initiative power reserved by the people by amendment to the A system of incentives for exceeding the set expectations of a public office
Constitution x x x applies only to the proposing and the adopting or rejecting of "laws is not anathema to the concept of public account ability
and amendments to the Constitution" and does not purport to extend to a constitutional
revision. It is a fundamental principle that a constitution can only be revised or amended Statements of Assets, Liabilities and Networth – Rabe vs. Flores
in the manner prescribed by the instrument itself, and that any attempt to revise a - An administrative complaint was filed against the respondent Flores for “conduct
constitution in a manner other than the one provided in the instrument is almost unbecoming a government employee, acts prejudicial to the interest of the service
invariably treated... as extra-constitutional and revolutionary. A change in the structure and abuse of authority.” It was alleged that Flores took advantage of her position
of government is a revision of the Constitution, as when the three great co-equal as a court employee by claiming a stall at the public market when she was not
branches of government in the present Constitution are reduced into two. This alters awarded the market stall by the court in a civil case. She had also failed to report
the separation of powers in the Constitution. A shift from the present. Bicameral- her business interest, and was found guilty for dishonesty wich resulted to her
Presidential system to a Unicameral-Parliamentary system is a revision of the dismissal.
Constitution
- The failure of respondent to disclose her business interest which she herself
- “Amendment” and “Revision,” Distinguished; Words and Phrases; The framers admitted is inexcusable and is a clear violation of RA 6713.
of the Constitution intended, and wrote, a clear distinction between
“amendment” and “revision” of the Constitution.—The framers intended, and LAW DEFINES POWER. NO OFFICIAL, NO MATTER HOW HIGH, IS ABOVE THE LAW.
wrote, that a people’s initiative may propose only amendments to the - Villavicencio vs. Lukban
Constitution. Where the intent and language of the Constitution clearly withhold
from the people the power to propose revisions to the Constitution, the people - Justo Lukban as Manila City's Mayor together with Anton Hohmann, the city's Chief
cannot propose revisions even as they are empowered to propose amendments. of Police, took custody of about 170 women at the night of October 25 beyond the
latter’s consent and knowledge and thereafter were shipped to Mindanao
specifically in Davao where they were signed as laborers. Said women are inmates
of the houses of prostitution situated in Gardenia Street, in the district of Sampaloc.
When the families of these women filed for habeas corpus, the respondent moved
CONSTI REVIEWER 26

to dismiss the case saying that those women were already out of their jurisdiction -It is noted by the Supreme Court that it cannot expand its judicial review of the actions of the
and that, it should be filed in the city of Davao instead. Senator-Judges on an impeachment trial as they are clearly entitled to ask questions on the
- The court ruled that we are clearly a government of laws. Lukban committed a grave witnesses, prosecutors, and counsel during such trial. Petition for certiorari was DISMISSED
abuse of discretion by deporting the prostitutes to a new domicile against their will. on the ground of MOOTNESS.
There is no law expressly authorizing his action. On the contrary, there is a law
punishing public officials who not expressly authorized by law or regulation, -Impeachment refers to the power of Congress to remove a public official for serious
compels any person to change his residence. Furthermore, the prostitutes are still, crimes or misconduct as provided in the Constitution. Impeachment was also
as citizens of the Philippines, entitled to the same rights, as stipulated in the Bill of described as “the most formidable weapon in the arsenal of democracy”
Rights, as every other citizen. Their choice of profession should not be a cause for IMPEACHMENT
discrimination. It may make some, like Lukban, quite uncomfortable but it does not Estrada vs. Desierto, GR 146710-15, March 2, 2001
authorize anyone to compel said prostitutes to isolate themselves from the rest of
the human race. These women have been deprived of their liberty by being exiled Following accusations of corruption, President Joseph Estrada had been called to resign. A
to Davao without even being given the opportunity to collect their belongings or, trial for his impeachment also began years after; which was postponed upon Chief Justice
worse, without even consenting to being transported to Mindanao. Davide’s grant of Senator Raul Roco’s motion. Still calls for the President’s resignation
persisted and news broke out that Chief Justice Hilario Davide would administer the oath of
PROCEDURE FOR IMPEACHMENT - FRANCISCO VS. HR presidency to VP Gloria Arroyo at EDSA Shrine. Estrada issued two statements - one stating
- In Francisco vs. HR, two impeachment complaints were filed against Chief Justice reservations on the constitutionality of Arroyo’s presidency, and another stating that he is
Hilario G. Davide Jr. and seven Associate Justices within one year despite the incapable of dispensing his responsibilities as president, thus allowing Arroyo to be the acting
constitutional prohibition of the same under Art. XI, Sec. 3 (5), which states that no president.
impeachment proceedings shall be initiated against the same official more than The impeachment trial was closed, despite sentiments such as those of Senator Defensor-
once within a period of one year. Said Constitutional prohibition is non-self- Santiago that the impeachment court had failed to resolve the case, leaving open questions
executing and thus requires congressional enactment. The 12th congress adopted regarding Estrada’s qualifications to run for other elected posts. Still, the Office of the
and approved the Rules of Procedure in Impeachment Proceedings the relevant Ombudsman proceeded to file a series of cases regarding the corruption of Estrada. Estrada
portion of which reads: impeachment proceedings are deemed initiated at the time filed a motion compelling the Ombudsman to refrain from further proceedings until his term
of the filing of such verified complaint or resolution of impeachment with the as president was over.
Secretary General.
- The court ruled that initiation takes place by the act of filing of the impeachment The Court ruled that petitioner cannot invoke double jeopardy. Double jeopardy attaches
complaint and referral to the House Committee on Justice. Rules and regulation only: (1) upon a valid complaint; (2) before a competent court; (3) after arraignment; (4) when
contrary to this are deemed unconsititutional. a valid plea has been entered; and (5) when the defendant was acquitted or convicted or the
case was dismissed or otherwise terminated without the express consent of the accused.
It is without a doubt that the term “to initiate” refers to the filing of the Assuming arguendo that the first four requisites of double jeopardy were complied with,
impeachment complaint coupled with Congress’ taking initial action of said petitioner failed to satisfy the fifth requisite for he was not acquitted nor was the impeachment
complaint; Once an impeachment complaint has been initiated, another proceeding dismissed without his express consent. Petitioner’s claim of double jeopardy
impeachment complaint may not be filed against the same official within a cannot be predicated on prior conviction for he was not convicted by the impeachment court.
one year period At best, his claim of previous acquittal may be scrutinized in light of a violation of his right to
IMPEACHMENT – CORONA V. SENATE speedy trial, which amounts to a failure to prosecute. As Bernas points out, a failure to
prosecute, which is what happens when the accused is not given a speedy trial, means failure
- The petitioner on his impeachment raised his concerns with regard to a political of the prosecution to prove the case. Hence, dismissal on such grounds is a dismissal on the
agenda behind his ouster. Citing instances of the behavior of the President towards merits.
him and most importantly of certain Senator-Judges in the course of his trial.
Petitioner reiterates the argument that he must be first convicted in the impeachment
proceedings before he could be criminally prosecuted. A plain reading of the provision
CONSTI REVIEWER 27

will not yield this conclusion. The provision conveys two uncomplicated ideas: first, it done by the House Speaker alone either, which explains why there is a need to include
tells us that judgment in impeachment cases has a limited reach . . . i.e., it cannot it in the Order of Business of the House. It is the House of Representatives, in public
extend further than removal from office and disqualification to hold any office under plenary session, which has the power to set its own chamber into special operation
the Republic of the Philippines, and second, it tells us the consequence of the limited by referring the complaint or to otherwise guard against the initiation of a second
reach of a judgment in impeachment proceedings considering its nature, i.e., that the impeachment proceeding by rejecting a patently unconstitutional complaint.
party convicted shall still be liable and subject to prosecution, trial and punishment
according to law. No amount of manipulation will justify petitioner’s non sequitur
submission that the provision requires that his conviction in the impeachment ILL-GOTTEN WEALTH – PRESIDENTIAL AD HOC VS DISIERTO, GR 130140
proceedings is a condition sine qua non to his prosecution, trial and punishment for - Brief Summary: The case is about Presidential Ad Hoc Fact-finding Committee on
the offenses he is now facing before the respondent OmbudsmanRA 2630- BY Behest Loans assailing the decision of Ombudsman Disierto holding that the offenses
STATUS QUO IMPEACHMENT-Guiterrez vs HR Case committed in relation to the behest loans have already prescribed.
- Ruling: Behest loans are part of the ill-gotten wealth which former President Marcos
- Baraquel, et.al. filed an impeachment complaint against petitioner Ombudsman Ma. and his cronies accumulated and which the government through the PCGG seeks to
Merceditas N. Guiterrez based on betrayal of public trust and culpable violation of the recover. The Court ruled that if the commission of the crime is known, the prescriptive
Constitution. Reyes et.al. filed another impeachment complaint against the same on the period shall commence to run on the day it was committed, however, it was well-nigh
same grounds. The Court voted to issue a status quo ante order suspending the impossible for the State, the aggrieved party, to have known the violations of RA No.
impeachment proceedings against petitioner. 3019 at the time the questioned transactions were made because at alleged, the public
officials concerned connived or conspired with the beneficiaries of the loans. Thus, the
- The Court, of course, does not downplay the importance of an impeachment complaint, for prescription period for the offenses committed should be computed from discovery of
it is the matchstick that kindles the candle of impeachment proceedings. The filing of an the commission and not from the day of such commission.
impeachment complaint is like the lighting of a matchstick. Lighting the matchstick alone, - Jurisprudence: Section 15 of Article XI of the Constitution states that “The right of
however, cannot light up the candle, unless the lighted matchstick reaches or torches the the State to recover properties unlawfully acquired by public officials or
candle wick. Referring the complaint to the proper committee ignites the impeachment employees shall not be barred by prescription”.
proceeding. With a simultaneous referral of multiple complaints filed, more than one lighted
matchsticks light the candle at the same time. What is important is that there should only Khan vs. Ombudsman
be ONE CANDLE that is kindled in a year, such that once the candle starts burning,
subsequent matchsticks can no longer rekindle the candle. - Private respondents Rosauro Torralba and Celestino Bandala charged petitioners
before the Deputy Ombudsman (Visayas) for violation of RA 3019 (The Anti-Graft
and Corrupt Practices Act). They accused petitioners of using their positions in
- The question as to who should administer or pronounce that an impeachment proceeding Philippine Airlines (PAL) to secure a contract for Synergy Services Corporation, a
has been initiated rests also on the body that administers the proceedings prior to the corporation engaged in hauling and janitorial services in which they were
impeachment trial. As gathered from Commissioner Bernas' disquisition[76] in Francisco, shareholders. Petitioners filed an omnibus motion to dismiss the complaint on the
a proceeding which "takes place not in the Senate but in the House"[77] precedes the bringing following grounds: (1) the Ombudsman had no jurisdiction over them since PAL
of an impeachment case to the Senate. In fact, petitioner concedes that the initiation of was a private entity and (2) they were not public officers, hence, outside the
impeachment proceedings is within the sole and absolute control of the House of application of RA 3019.
Representatives.[78] Conscious of the legal import of each step, the House, in taking charge - The Court ruled that public respondents Deputy Ombudsman have no jurisdiction
of its own proceedings, must deliberately decide to initiate an impeachment proceeding, over petitioners because pursuant to Article XI, Section 13(2) of the 1987
subject to the time frame and other limitations imposed by the Constitution. This chamber of Constitution, it specifically states the powers and function of the Office of the
Congress alone, not its officers or members or any private individual, should own up to its Ombudsman.
processes. - “The Office of the Ombudsman shall have the power, to direct, upon
- The Constitution did not place the power of the "final say" on the lips of the House complaint or at its own instance, any government-owned and controlled
Secretary General who would otherwise be calling the shots in forwarding or freezing corporation with original charter, to perform and expedite any act or duty
any impeachment complaint. Referral of the complaint to the proper committee is not
CONSTI REVIEWER 28

required by law, or to stop, prevent, and correct any abuse or impropriety in - The Constitution and RA No. 6770 endowed the Office of the Ombudsman with
the performance of duties”. wide latitude, in the exercise of its investigatory and prosecutory powers, to pass
upon criminal complaints involving public officials and employees. Specifically, the
The Office of the Ombudsman exercises jurisdiction over public officials/employees of the determination whether probable cause exists is a function that belongs to the Office
government corporations, although the government later on acquired the controlling interest of the Ombudsman. The Supreme Court must recognize the decision of the
in PAL, it did not have an “original charter” and its officers/employees could not be Ombudsman as when the SC can not determine the criminal liability in this case,
investigated and/or prosecuted by the Ombudsman as it would violate the Constitutional and Legislative powers granted to the Office
of the Ombudsman, as well as the Constitutional delineation of powers.

OMBUDSMAN) OMBUDSMAN VS. CA - Ombudsman; Policy of Noninterference; As a general rule, the Supreme Court
(SC) does not intervene with the Ombudsman’s exercise of its investigative
- Joan and Thomas Corominas, and Maria Constancia Corominas-Lim filed with the and prosecutorial powers, and respects the initiative and independence
Office of the Ombudsman (Visayas) a criminal complaint for violation of Article 281 inherent in the Office of the Ombudsman which, beholden to no one, acts as
(Other Forms of Trespass) of the P|RPC against respondents who are employees the champion of the people and the preserver of the integrity of the public
of the Department of Environment and Natural Resources (DENR), Regional Office service. — This policy rests on the fundamental doctrine of separation of powers,
No. VII, Banilad, Mandaue City. It was alleged that the DENR employees conspired which is one of the foundations of our republican government.
to enter the parcel of land owned by the Corominas family without seeking
permission from the latter or their representative and despite the big "NO
TRESPASSING" sign attached to the perimeter fences enclosing the said property. Same; The 1987 Constitution clothed the Ombudsman with authority to investigate
- The Court rejected the argument that the power of the Office of the Ombudsman is offenses committed by public officers and employees. — The 1987 Constitution clothed
only advisory or recommendatory in nature rather, it is mandatory within the bounds the Ombudsman with authority to investigate offenses committed by public officers and
of R.A. 6770 which is enacted to provide for the functional and structural employees. Specifically, the determination of whether probable cause exists is a function that
organization of the function of the Office of the Ombudsman which includes. belongs to the Office of the Ombudsman. Whether a criminal case, given its attendant facts
and circumstances, should be filed or not is basically its call.
- Ombudsman; The mandatory character of the Ombudsman’s order imposing
a sanction should not be interpreted as a usurpation of the authority of the
head of office or any officer concerned because its power to investigate and
prosecute any illegal act or omission of any public official is not an exclusive
authority but a shared or concurrent authority in respect of the offense
charged.
Ombudsman – ITF vs. Comelec; G.R. No. 159139 January 13, 2004
- COMELEC awarded a contract for the procurement of automated counting
machines (ACMs) for the national elections to Mega Pacific Consortium (MPC).
Later, the SC found that MPC failed to comply with the essential requirements of a
public bidding as well as a possible grave abuse of discretion on the part of
COMELEC. The SC declared the contract with MPC as null and void, and directed
the Ombudsman to determine if the public officials involved in the case are
criminally liable. After initiating a fact-finding investigation and after numerous
procedures, reversed the SC directive and dismissed the charges against MPC and
the involved public officials for lack of probable cause.

Das könnte Ihnen auch gefallen